Women's Health Exam 2 [Monday 3/6/17]

Réussis tes devoirs et examens dès maintenant avec Quizwiz!

*mgmt of Disseminated Intravascular Coagulation (DIC)*

Continue to provide nursing care related to the triggering event. Assess the child's status frequently. If bleeding is observed, apply pressure to the area along with cold compresses. Elevate the affected body part if this does not affect the child's overall stability. If neurologic deficits are assessed, report the findings immediately so that treatment to prevent permanent damage can be started. Administer anticoagulation therapy (even though hemorrhage is a concern) to interrupt the coagulation process that is present in this condition. Provide ventilatory support as needed and provide continuous cardiac monitoring. Administer clotting factors, platelets, and cryoprecipitate as prescribed to prevent severe hemorrhage. Report changes in laboratory values to the physician or nurse practitioner. Changes can occur rapidly, and vigilance is necessary to prevent further tissue damage to the affected system.

*what a molar pregnancy is*

- Hydatidiform mole is a benign neoplasm of the chorion in which the chorionic villi degenerate and become transparent vesicles containing clear, viscid fluid - is classified as complete or partial, distinguished by differences in clinical presentation, pathology, genetics, and epidemiology - The complete mole contains no fetal tissue and develops from an "empty egg," which is fertilized by a normal sperm (the paternal chromosomes replicate, resulting in 46 all-paternal chromosomes) - The embryo is not viable and dies - No circulation is established, and no embryonic tissue is found - The complete mole is associated with the development of choriocarcinoma. Surgery can totally remove most complete moles, but as many as one in five women will have some persistent molar tissue and require further treatment - Most women with a classic complete mole present with vaginal bleeding, anemia, excessively enlarged uterus, preeclampsia, and hyperemesis. - The partial mole has a triploid karyotype (69 chromosomes), because two sperm have provided a double contribution by fertilizing the ovum. Women with a partial mole usually present with the clinical features of a missed or incomplete abortion, including vaginal bleeding and a small or normal size for date uterus - The exact cause of molar pregnancy is unknown, but researchers are looking into a genetic basis, Other theories include an ovular defect, stress, or a nutritional deficiency (carotene), Although the etiology remains uncertain, at some point in the pregnancy trophoblastic cells that normally would form the placenta proliferate and the chorionic villi become edematous - The latter changes become the grape-like clusters that characterize the molar pregnancy - studies have revealed some remarkable features about molar pregnancies, including: Ability to invade into the wall of the uterus Tendency to recur in subsequent pregnancies Possible development into choriocarcinoma, a virulent cancer with metastasis to other organs Influence of nutritional factors, such as protein deficiency Tendency to affect older women more often than younger women Having a molar pregnancy (partial or complete) results in the loss of the pregnancy and the possibility of developing choriocarcinoma, a chorionic malignancy from the trophoblastic tissue. Typically asymptomatic, the first symptoms of choriocarcinoma in 80% of cases are shortness of breath, indicative of metastasis to the lungs. Choriocarcinoma affects women of all ages and can occur during pregnancy, after childbirth, or even years remote from the antecedent pregnancy. The most frequent sites of metastases are the lungs, lower genital tract, brain, and liver. Choriocarcinoma is highly responsive to chemotherapy, with an overall remission rate greater than 90% (Hensley & Shviraga, 2014). Partial moles rarely transform into choriocarcinoma.

Habitual abortion

*Hx of 3 or more consecutive spontaneous abortions* Not carrying the pregnancy to viability or term Validation via client's history - mgmt: *Identification and treatment of underlying cause (possible causes such as genetic or chromosomal abnormalities, reproductive tract abnormalities, chronic diseases, or immunologic problems), Cervical cerclage in second trimester if incompetent cervix is the cause*

*landmarks*

*Occipital bone (0)*: vertex presentation/position *Chin (mentum [M])*: face presentation C section required *Buttocks (sacrum [S])*: breech presentation Breach position *Scapula (acromion process [A])*: shoulder presentation

*mgmt of hyperemesis gravidarum*

*Priority Interventions: it's is nausea and vomiting in early pregnancy that prevents the woman from ingesting adequate nutrition. IV fluids may be required for rehydration, but the priority is to stop all intake of food and fluid for a period of time until vomiting has stopped.* - Hyperemesis gravidarum is a diagnosis of exclusion. Careful consideration of other conditions must be assessed when a client experiences nausea and vomiting for the first time after 9 weeks' gestation. Conservative management in the home is the first line of treatment for the woman with hyperemesis gravidarum. This usually focuses on dietary and lifestyle changes. If conservative management fails to alleviate the client's symptoms and nausea and vomiting continue, hospitalization is necessary to reverse the effects of severe nausea and vomiting. On admission to the hospital, blood tests are ordered to assess the severity of the client's dehydration, electrolyte imbalance, ketosis, and malnutrition. Parenteral fluids and drugs are ordered to rehydrate the woman and reduce the symptoms. The first choice for fluid replacement is generally normal saline, which aids in preventing hyponatremia, with vitamins (pyridoxine [B6]) and electrolytes added. Oral food and fluids are withheld for the first 24 to 36 hours to allow the gastrointestinal tract to rest. Antiemetics may be administered rectally or intravenously to control the nausea and vomiting initially because the woman is considered NPO (not able to ingest anything by mouth). Once her condition stabilizes and she is allowed oral intake, medications may be administered orally. If the client does not improve after several days of bed rest, "gut rest," IV fluids, and antiemetics, total parenteral nutrition or feeding through a percutaneous endoscopic gastrostomy tube is instituted to prevent malnutrition. Administering antiemetics intravenously or intramuscularly is typically the second pillar of treatment for hyperemesis gravidarum. Finding a drug that works for any given client is largely a matter of trial and error. If one drug is ineffective, another class of drugs with a different mechanism of action may help. Promethazine (Phenergan) and prochlorperazine (Compazine) are among the older preparations usually tried first. If they fail to relieve symptoms, newer drugs such as ondansetron (Zofran) may be tried. Most drugs are given intravenously or intramuscularly. There is no evidence that any antiemetic class is superior to another with respect to effectiveness - Few women receive complete relief of symptoms from any one therapy. Complementary and alternative medicine therapies appeal to many women as supplements to traditional ones. Some popular therapies include acupressure, massage, therapeutic touch, ginger, and the wearing of Sea-Bands to prevent nausea and vomiting. Recent research has reported a positive effect of using acupressure (provided by Sea-Bands) over the nei guan acupoint on the wrist to control nausea and vomiting associated with pregnancy

*assessment findings of Disseminated Intravascular Coagulation (DIC)*

*Take Note! When excessive bleeding occurs during the childbirth process and it persists or signs such as bruising or petechiae appear, disseminated intravascular coagulation (DIC) should be suspected.* - *DIC Diagnostic Testing: Diagnostic tests that indicate the development of DIC include increased fibrinogen/fibrin degradation products, decreased antithrombin III, increased fibrinopeptide A level, and an increased D-dimer assay.* - Because DIC occurs as a secondary condition, it may occur in a child hospitalized for any reason. DIC may affect any body system, so a thorough physical examination is warranted. Inspect for signs of bleeding such as petechiae or purpura, blood in the urine or stool, or persistent oozing from venipuncture or from the umbilical cord in the newborn. Evaluate respiratory status and determine the level of tissue oxygenation via pulse oximetry. Perform a complete circulatory assessment and note signs of circulatory collapse such as poor perfusion, tachycardia, prolonged capillary refill, and weak distal pulses. Note altered level of consciousness and decreased urine output. Careful abdominal palpation may reveal hepatomegaly or splenomegaly. Laboratory testing may reveal prolonged prothrombin time (PT), partial thromboplastin time (PTT), activated partial thromboplastin time (aPTT), bleeding time, and thrombin time and decreased levels of fibrinogen; platelets; clotting factors II, V, VIII, and X; and antithrombin III. Increases will be noted in levels of fibrinolysin, fibrinopeptide A, positive fibrin split products, and D-dimers.

*Inevitable abortion*

*Vaginal bleeding (greater than that associated with threatened abortion) [mod-heavy]* Rupture of membranes Cervical dilation Strong abdominal cramping *Open cervical os* Possible passage of products of conception Ultrasound and hCG levels to indicate pregnancy loss - mgmt: *Vacuum curettage if products of conception are not passed, to reduce risk of excessive bleeding and infection, Prostaglandin analogs such as misoprostol to empty uterus of retained tissue (only used if fragments are not completely passed)*

*Threatened abortion*

*Vaginal bleeding (often slight) early in a pregnancy [spotting]* No cervical dilation or change in cervical consistency *Mild/slight abdominal/uterine cramping* *Closed cervical os* No passage of fetal tissue Vaginal ultrasound to confirm if sac is empty Declining maternal serum hCG and progesterone levels to provide additional information about viability of pregnancy - mgmt: *Conservative supportive treatment, Possible reduction in activity in conjunction with nutritious diet and adequate hydration*

While waiting for the placenta to deliver during the third stage of labor the nurse must assess the new mother's vital signs every 15 minutes. What sign would indicate impending shock? a. tachypnea and a widening pulse pressure b. tachycardia and a falling blood pressure c. bradycardia and auscultation of fluid in the base of the lungs d. bradypnea and hypertension

*b. tachycardia and a falling blood pressure*

*clinical manifestations of placenta previa*

- Ask the client if she has any problems associated with bleeding, now or in the recent past. The classical clinical presentation is painless, bright-red vaginal bleeding occurring during the second or third trimester. The initial bleeding usually is not profuse and it ceases spontaneously, only to recur again. The first episode of bleeding occurs (on average) at 27 to 32 weeks' gestation. The bleeding is thought to arise secondary to the thinning of the lower uterine segment in preparation for the onset of labor. When the bleeding occurs at the implantation site in the lower uterus, the uterus cannot contract adequately and stop the flow of blood from the open vessels. Typically with normal placental implantation in the upper uterus, minor disruptive placental attachment is not a problem, because there is a larger volume of myometrial tissue able to contract and constrict bleeding vessels. Assess the client for uterine contractions, which may or may not occur with the bleeding. Palpate the uterus; typically it is soft and nontender on examination. Auscultate the fetal heart rate; it commonly is within normal parameters. Fetal distress is usually absent but may occur when cord problems arise, such as umbilical cord prolapse or cord compression, or when the client has experienced blood loss to the extent that maternal shock or placental abruption has occurred - To validate the position of the placenta, a transvaginal ultrasound is done. In addition, magnetic resonance imaging (MRI) may be ordered when preparing for delivery because it allows identification of placenta accreta (placenta abnormally adherent to the myometrium), increta (placenta accreta with penetration of the myometrium), or percreta (placenta accreta with invasion of the myometrium to the peritoneal covering, causing rupture of the uterus) in addition to placenta previa. These placental abnormalities, although rare, carry a very high morbidity and mortality rate, possibly necessitating a hysterectomy at delivery.

*clinical manifestations of hyperemesis gravidarum*

- Begin the history by asking the client about the onset, duration, and course of her nausea and vomiting. Ask her about any medications or treatments she used and how effective they were in relieving her nausea and vomiting. Obtain a diet history from the client, including a dietary recall for the past week. Note the client's knowledge of nutrition and need for appropriate nutritional intake. Be alert for patterns that may contribute to or trigger her distress. Also ask about any complaints of ptyalism (excessive salivation), anorexia, indigestion, and abdominal pain or distention. Ask if she has noticed any blood or mucus in her stool. Review the client's history for possible risk factors, such as young age, nausea and vomiting with previous pregnancy, history of intolerance of oral contraceptives, nulliparity, trophoblastic disease, multiple gestation, emotional or psychological stress, gastroesophageal reflux disease, primigravida status, obesity, hyperthyroidism, and Helicobacter pylori seropositivity. Weigh the client and compare this weight with her weight before she began experiencing symptoms and to her prepregnancy weight to estimate the degree of loss. With hyperemesis, weight loss usually exceeds 5% of body mass. Inspect the mucous membranes for dryness and check skin turgor for evidence of fluid loss and dehydration. Assess blood pressure for changes, such as hypotension, that may suggest a fluid volume deficit. Also note any complaints of weakness, fatigue, activity intolerance, dizziness, or sleep disturbances. Assess the client's perception of the situation. Note any evidence of depression, anxiety, irritability, mood changes, and decreased ability to concentrate, which can add to her emotional distress. Much of the psychological distress is self-limiting in this condition and probably in the causal pathway. Determine the woman's support systems that are available for help. - Liver enzymes—to rule out hepatitis, pancreatitis, and cholestasis; elevations of aspartate aminotransferase (AST) and alanine aminotransferase (ALT) are usually present CBC—elevated levels of red blood cells and hematocrit, indicating dehydration Urine ketones—positive when the body breaks down fat to provide energy in the absence of adequate intake TSH and T4 to rule out thyroid disease Blood urea nitrogen—increased in the presence of salt and water depletion Urine specific gravity—greater than 1.025, possibly indicating concentrated urine linked to inadequate fluid intake or excessive fluid loss; ketonuria Serum electrolytes—decreased levels of potassium, sodium, and chloride resulting from excessive vomiting and loss of hydrochloric acid in stomach Ultrasound—evaluation for molar pregnancy or multiple gestation

*nursing implications and precautions when administering Magnesium Sulfate*

- Blockage of neuromuscular transmission, vasodilation - Prevention and treatment of eclamptic seizures - Nursing implications: Loading dose of 4-6 g by IV in 100 mL of fluid administered over 15-20 minutes, followed by a maintenance dose of 2 g as a continuous IV infusion. Monitor serum magnesium levels closely. Assess DTRs and check for ankle clonus. *Have calcium gluconate readily available in case of toxicity.* *Monitor for signs and symptoms of toxicity: flushing, sweating, hypotension, and cardiac and central nervous system depression.*

*Nursing management for the client with hyperemesis gravidarum* focuses on promoting comfort by controlling the client's nausea and vomiting and promoting adequate nutrition. In addition, the nurse plays a major role in supporting and educating the client and her family.

- During the initial period, expect to withhold all oral food and fluids, maintaining NPO status to allow the gastrointestinal tract to rest. In addition, administer prescribed antiemetics to relieve the nausea and vomiting and IV fluids to replace fluid losses. Monitor the rate of infusion to prevent overload and assess the IV insertion site to prevent infiltration or infection. Also administer electrolyte replacement therapy as ordered to correct any imbalances, and periodically check serum electrolyte levels to evaluate the effectiveness of therapy. Provide physical comfort measures such as hygiene measures and oral care. Pay special attention to the environment, making sure to keep the area free of pungent odors. As the client's nausea and vomiting subside, gradually introduce oral fluids and foods in small amounts. Monitor intake and output and assess the client's tolerance to the increase in intake. - Women with hyperemesis gravidarum commonly are fatigued physically and emotionally. Many are exhausted, frustrated, and anxious. Offer reassurance that all interventions are directed toward promoting positive pregnancy outcomes for both the woman and her fetus. Providing information about the expected plan of care may help to alleviate the client's anxiety. Listen to her concerns and feelings, answering all questions honestly. Educate the woman and her family about the condition and its treatment options (Teaching Guidelines 19.1). Teach the client about therapeutic lifestyle changes, such as avoiding stressors and fatigue that may trigger nausea and vomiting. Offer ongoing support and encouragement and promote active participation in care decisions, thereby empowering the client and her family. Attempting to provide the client with a sense of control may help her overcome the feeling that she has lost control. If necessary, refer the client to a spiritual advisor or counseling. Also suggest possible local or national support groups that the client may contact for additional information. Arrange for possible home care follow-up for the client and reinforce discharge instructions to promote understanding. Timely counseling, balanced nutrition, pharmacotherapy, and emotional support are associated with favorable outcomes for the woman with this condition. Collaborate with community resources to ensure continuity of care.

*mgmt of ectopic pregnancy*

- Historically, the treatment of ectopic pregnancy was limited to surgery. - The therapeutic management of ectopic pregnancy depends on whether the tube is intact or has ruptured, creating a medical emergency. - In the event of a surgical intervention, preservation of the affected fallopian tube is attempted - Screen serum progesterone and B-hCG - Assess for active bleeding → rupture May have vertigo, shoulder pain, hypotension & tachycardia - Salpingostomy can be done before rupture - Methotrexate after surgery to dissolve residual tissue - Laporotomy if pregnancy advanced

*clinical manifestations of ectopic pregnancy* Nursing assessment focuses on determining the existence of an ectopic pregnancy and whether or not it has ruptured. A woman with a suspected ectopic pregnancy may have to undergo several diagnostic tests, some of which are invasive. - The diagnosis of ectopic pregnancy can be challenging because many women are asymptomatic before tubal rupture. - The classic clinical triad of ectopic pregnancy includes *abdominal pain, amenorrhea, and vaginal bleeding*. Unfortunately, only about 1/2 of women present with all 3 symptoms.

- Take Note!*The hallmark of ectopic pregnancy is abdominal pain with spotting within 6 to 8 weeks after a missed menstrual period. Although this is the classic triad, all three of these signs and symptoms occur in only about 50% of cases. Many women have symptoms typical of early pregnancy, such as breast tenderness, nausea, fatigue, shoulder pain, and low back pain.* - the onset of signs and symptoms usually begin at about the 7th or 8th week of gestation 1. A missed menstrual period 2. adnexal fullness, and tenderness may indicate an unruptured tubal pregnancy, as the tube stretches, the pain increases, pain may be unilateral, bilateral, or diffuse over the abdomen [dull-colicky] 3. 50%-80% have dark red or brown vaginal bleeding 4. May have shoulder pain from diaphragmatic irritation caused by blood in peritoneal cavity 5. may have signs of shock 6. *Blueness around umbilicus - Cullen Sign - hematoperitoneum* - Suspect EP if woman presents with abd pain, vaginal spotting/bleeding and pos preg test - if rupture or hemorrhage occurs before treatment begins (active bleeding), symptoms may worsen and include *severe, sharp, and sudden pain in the lower abdomen as the tube tears open and the embryo is expelled into the pelvic cavity; feelings of faintness (vertigo); referred pain to the shoulder area, indicating bleeding into the abdomen, caused by phrenic nerve irritation; hypotension; marked abdominal tenderness with distention; and hypovolemic shock*

*possible sites for implantation w/ an ectopic pregnancy*

- cervix - fallopian tube - ovary - intestine - abdomen - Normally, the fertilized ovum implants in the uterus. In ectopic pregnancy the journey along the fallopian tube is arrested or altered in some way. With an ectopic pregnancy, the ovum implants outside the uterus. The most common site for implantation is the fallopian tubes, but some ova may implant in the ovary, the intestine, the cervix, or the abdominal cavity. None of these anatomic sites can accommodate placental attachment or a growing embryo.

*mgmt of placenta previa*

-*Take Note! Avoid doing vaginal examinations in the woman with placenta previa because they may disrupt the placenta and cause hemorrhage.* - Therapeutic management depends on the extent of bleeding, the amount of placenta over the cervical os, whether the fetus is developed enough to survive outside the uterus, the position of the fetus, the mother's parity, and the presence or absence of labor. With the increase in the rate of previous cesarean sections, the frequency of placenta previa has increased. Most women continue to present in emergency departments, therefore the associated morbidity due to hemorrhage remains high. Efforts should be made to avoid primary cesarean section where possible. In addition, prenatal care and timely diagnosis of placenta previa on ultrasound can decrease the associated morbidity. If the mother and fetus are both stable, therapeutic management may involve expectant ("wait-and-see" or watchful waiting) care. This care can be carried out at home or on an antepartal unit in the health care facility. If there is no active bleeding and the client has ready access to reliable transportation, can maintain bed rest at home, and can comprehend instructions, expectant care at home is appropriate. However, if the client requires continuous care and monitoring and cannot meet the home care requirements, the antepartal unit is the best environment. - Whether the care setting is in the client's home or in the health care facility, the nurse focuses on monitoring the maternal-fetal status, including assessing for signs and symptoms of vaginal bleeding and fetal distress and providing support and education to the client and her family, including providing information about the diagnostic studies and procedures that are performed. For the majority of women, a cesarean birth will be planned. - Assess the degree of vaginal bleeding; inspect the perineal area for blood that may be pooled underneath the woman. Estimate and document the amount of bleeding. Perform a peripad count on an ongoing basis, making sure to report any changes in amount or frequency to the health care provider. If the woman is experiencing active bleeding, prepare for blood typing and cross-matching in the event a blood transfusion is needed. - Monitor maternal vital signs and uterine contractility frequently for changes. Have the client rate her level of pain using an appropriate pain rating scale. Assess fetal heart rates via Doppler or electronic monitoring to detect fetal distress. Monitor the woman's cardiopulmonary status, reporting any difficulties in respirations, changes in skin color, or complaints of difficulty breathing. Have oxygen equipment readily available should fetal or maternal distress develop. Encourage the client to lie on her side to enhance placental perfusion. If the woman has an intravenous (IV) line inserted, inspect the IV site frequently. Alternately, anticipate the insertion of an intermittent IV access device such as a saline lock, which can be used if quick access is needed for fluid restoration and infusion of blood products. Obtain laboratory tests as ordered, including complete blood count (CBC), coagulation studies, and Rh status if appropriate. Administer pharmacologic agents as necessary. Give Rh immunoglobulin if the client is Rh negative at 28 weeks' gestation. Monitor tocolytic (anticontraction) medication if prevention of preterm labor is needed.

*s/s of magnesium toxicity*

1. *Decreased DTR's!!!* 2. Resp. depression 3. Oliguria 4. Decreased LOC 5. slurred speech 6. Nausea 7. Feeling of warmth 8. *Flushing (teach that flu symptoms are supposed to happen/common)* 9. Muscle weakness Antidote: *calcium gluconate* (always make sure it is available)

*understand what the 5 "P" factors are, the effect they have on the labor process, and the nursing intervention related to each* At least 5 factors affect process of labor and birth. 5 P's:

1. Passageway (birth canal) 2. Passenger (fetus and placenta) 4. Powers (contractions, *pressure*) 4. Position (maternal) 5. Psychologic response (What kind of mood is she in)

*Which of the following tells us she wont be able to vaginal delivery? She had a c section w/ 1st baby*

1st baby breach - yes! 1st baby placental previa - yes! 1st baby had fetal distress - yes! 1st baby cephalopathic disproportion - no vaginal delivery, must have c section bc of odd shaped pelvis

nurse midwife

A nurse who completes a program in midwifery; provides prenatal and postnatal care; and delivers babies for women with uncomplicated pregnancies

*miscarriage (spontaneous abortion)*

A stillbirth is the loss of a fetus after the 20th week of development, whereas a miscarriage *refers to a loss before the 20th week or fetus is 500 g* - A miscarriage can occur during early pregnancy, and many women who miscarry may not even be aware that they are pregnant - About 80% of spontaneous abortions occur within the 1st trimester. - stillbirth and miscarriage both refer to the loss of an early pregnancy; however, stillbirth occurs later in pregnancy - Some stillbirths can occur right up to the time of labor and delivery, Stillbirths are much less common than miscarriages, occurring in only 1 out of every 160 pregnancies - Early is before 12 weeks - Late: between 12-20 wks - 50% of all miscarriages from chromosome abnormalities - Clinical manifestations: Uterine bleeding Uterine contractions Uterine pain *Before 6th week - may be like heavy period (more like straight up blood) *6th to 12th week: moderate discomfort & blood loss (like contraction pain)* *After 12 weeks: more severe pain like labor*

*Missed abortion* (nonviable embryo retained in utero for *at least 6 wks*)

Absent uterine contractions Irregular spotting Possible progression to inevitable abortion Closed cervical os *Ultrasound to identify products of conception in uterus* - mgmt: *Evacuation of uterus (if inevitable abortion does not occur): suction curettage during first trimester, dilation and evacuation during second trimester, Induction of labor with intravaginal PGE2 suppository to empty uterus without surgical intervention*

On the fetal heart monitor, the nurse notices an elevation of the fetal baseline with the onset of contractions. This elevation would describe _____________.

Acceleration—elevation of FHR above the baseline; a category I pattern, which is normal.

*patient education for molar pregnancy*

After GTD is diagnosed, teach the client about the condition and appropriate interventions that may be necessary to save her life. Explain each phase of treatment accurately and provide support for the woman and her family as they go through the grieving process.

*providing follow-up care for preeclampsia*

After delivery of the newborn, continue to monitor the client for signs and symptoms of preeclampsia/eclampsia for at least 48 hours. Expect to continue to administer magnesium sulfate infusion for 24 hours to prevent seizure activity, and monitor serum magnesium levels for toxicity. Assess vital signs at least every 4 hours, along with routine postpartum assessments: fundus, lochia, breasts, bladder, bowels, and the woman's emotional state. Monitor urine output closely. Diuresis is a positive sign that, along with a decrease in proteinuria, signals resolution of the disease.

*refusal of medical treatment*

All clients have the right to refuse medical treatment, based on the American Hospital Association's Bill of Rights - Ideally, medical care without informed consent should be used only when the client's life is in danger. Clients may refuse treatment if it conflicts with their religious or cultural beliefs (Brudney & Lantos, 2011). An example would be a Jehovah's Witness. Individuals of this faith have strong beliefs based upon passages from the Bible that are interpreted as prohibiting the "consumption" of blood. Their beliefs prevent them from accepting transfusion of whole blood or its primary components. With recent advances and use of biologic hemostats that aid coagulation and reduce blood loss, major surgery can be performed safely in the Jehovah's Witness who refuses blood transfusion by utilizing these devices (Darwish, 2011). In these cases, it is important to educate the client and family about the importance of the recommended treatment without coercing or forcing the client to agree. Sometimes common ground may be reached between the family's religious or cultural beliefs and the health care team's recommendations. Communication and education are the keys in this situation. In the case of a child, parental autonomy (the right to decide for or against medical treatment) is a fundamental, constitutionally protected right, but not an absolute one. The general assumption is that parents act in the best interests of their children. Ideally, medical care without informed consent should be given only when the child's life is in danger. In some cases parents may refuse medical treatment for their child. This refusal may arise when treatment conflicts with their religious or cultural beliefs, and the nurse should be aware of some of these common beliefs. Some religions, such as Christian Science, Pentecostal, Church of the First Born, and Followers of Christ, prefer prayer or faith healing over allopathic medicine (Bingham, 2012; Hall, 2013). The Black Muslim culture advocates vegan diets and refuses pork-based medicines or treatments. Hindus may refuse beef-based foods and medicines. People from an Islamic background may refuse the use of any potentially addictive substances such as narcotics or medicines containing alcohol (Leever, 2011). Sometimes common ground may be reached between the family's religious or cultural beliefs and the health care team's recommendations. - *Take Note! Do not assume what a family's beliefs are based on religious affiliation. Assess each family and child views on an individual basis.* In other cases, parents may refuse treatment if they perceive that their child's quality of life may be significantly impaired by the medical care that is offered. The health care team must appropriately educate the family and communicate with them on a level that they can understand. The child and family should be informed of what to expect with certain tests or treatments. The health care team should make a clinical assessment of the child's and family's understanding of the situation and their reasons for refusing treatment. Active listening may allow the health care provider to address the concerns, fears, or reservations the family may have regarding their child's care. Refusal of medical care may be considered a form of child neglect. If providing medical treatment may prevent substantial harm and suffering, or save a child's life, health care providers and the judicial system strive to advocate for the child. The state has an overriding interest in the health and welfare of the child and can order that medical treatment to proceed without signed informed consent; this is referred to as parens patriae (the state has a right and a duty to protect children). If the parents refuse treatment and the health care team feels the treatment is reasonable and warranted, the case should be referred to the institution's ethics committee. If the issue remains unresolved, or in complex cases, the judicial system may become involved

*follow-up care for molar pregnancy*

As with any facet of health care, be aware of the latest research and new therapies. Inform the client about her follow-up care, which will probably involve close clinical surveillance for approximately 1 year, and reinforce its importance in monitoring the client's condition. Tell the client that serial serum beta-hCG levels are used to detect residual trophoblastic tissue. Continued high or increasing hCG titers are abnormal and need further evaluation. Inform the client about the possible use of chemotherapy, such as methotrexate, which may be started prophylactically. Strongly urge the client to use a reliable contraceptive to prevent pregnancy for 1 year, because a pregnancy would interfere with tracking the serial beta-hCG levels used to identify a potential malignancy. Stress the need for the client to cooperate and adhere to the plan of therapy throughout this yearlong follow-up period.

*Teaching Guidelines 19.1 TEACHING TO MINIMIZE NAUSEA AND VOMITING*

Avoid noxious stimuli - such as strong flavors, perfumes, or strong odors such as frying bacon - that might trigger nausea and vomiting. Avoid tight waistbands to minimize pressure on abdomen. Eat small, frequent meals throughout the day—six small meals. Separate fluids from solids by consuming fluids in between meals. Avoid lying down or reclining for at least 2 hours after eating. Use high-protein supplement drinks. Avoid foods high in fat. Increase your intake of carbonated beverages. Increase your exposure to fresh air to improve symptoms. Eat when you are hungry, regardless of normal mealtimes. Drink herbal teas containing peppermint or ginger. Avoid fatigue and learn how to manage stress in life. Schedule daily rest periods to avoid becoming overtired. Eat foods that settle the stomach, such as dry crackers, toast, or soda.

BURP magnesium toxicity

BP decreased Urine output decreased RR <12 Patella reflex absent

*false labor*

Contractions -Occur irregularly or become regular only temporarily -Often stop with walking or position change -Can be felt in the back or abdomen above the navel. -Often can be stopped through the use of comfort measures Cervix: -May be soft, but there is no significant change in effacement or dilation or evidence of bloody show -Is often in a posterior position Fetus: -Presenting part is usually not engaged in pelvis

*true labor*

Contractions: - Occur regularly, become stronger, last longer, and occur closer together - Become more intense with walking. - Usually felt in lower back, radiating to lower portion of abdomen - Continue despite use of comfort measures (pain meds, etc) - *Will see visceral pain (low visceral abdominal pain)* Cervix: - Shows progressive change - Moves to an anterior position Fetus: - Presenting part usually becomes engaged in the pelvis

multiple gestation

Condition of bearing two or more fetuses simultaneously.

*mgmt for mild preeclampsia*

Conservative strategies for mild preeclampsia are used if the woman exhibits no signs of renal or hepatic dysfunction or coagulopathy. A woman with mild elevations in blood pressure may be placed on bed rest at home. She is encouraged to rest as much as possible in the lateral recumbent position to improve uteroplacental blood flow, reduce her blood pressure, and promote diuresis. In addition, antepartal visits and diagnostic testing - such as CBC, clotting studies, liver enzymes, and platelet levels - increase in frequency. The woman will be asked to monitor her blood pressure daily (every 4 to 6 hours while awake) and report any increased readings; she will also measure the amount of protein found in urine using a dipstick and will weigh herself for any weight gain. She also should take daily fetal movement counts, and if there is any decrease in movement, she needs to be evaluated by her health care provider that day. A balanced, nutritional diet with no sodium restriction is advised. In addition, she is encouraged to drink six to eight 8-oz glasses of water daily. If home management fails to reduce the blood pressure, admission to the hospital is warranted and the treatment strategy is individualized based on the severity of the condition and the gestational age at the time of diagnosis. During the hospitalization, the woman with mild preeclampsia is monitored closely for signs and symptoms of severe preeclampsia or impending eclampsia (e.g., persistent headache, hyperreflexia). Blood pressure measurements are frequently recorded along with daily weights to detect excessive weight gain resulting from edema. Fetal surveillance is instituted in the form of daily fetal movement counts, nonstress testing, and serial ultrasounds to evaluate fetal growth and amniotic fluid volume to confirm fetal well-being. Expectant management (watchful waiting) usually continues until the pregnancy reaches term, fetal lung maturity is documented, or complications develop that warrant immediate birth. Women with mild preeclampsia are at greatest risk for postpartum hypertension (King et al., 2015). Prevention of disease progression is the focus of treatment during labor. Blood pressure is monitored frequently and a quiet environment is important to minimize the risk of stimulation and to promote rest. IV magnesium sulfate is infused to prevent any seizure activity, along with antihypertensives if blood pressure values begin to rise. Calcium gluconate is kept at the bedside in case the magnesium level becomes toxic. Continued close monitoring of neurologic status is warranted to detect any signs or symptoms of hypoxemia, impending seizure activity, or increased intracranial pressure. An indwelling urinary (Foley) catheter usually is inserted to allow for accurate measurement of urine output.

The nurse is measuring a contraction from the beginning of the increment to the end of the decrement for the same contraction. The nurse would document this as which finding? a. Duration b. Intensity c. Frequency d. peak

Duration

At 32 weeks, a patient with hypertension since 28 weeks, hyperactive DTRs with clonus, and proteinuria of 4+ has a convulsion a. Eclampsia b. Gestational hypertension c. HELLP syndrome d. Severe Preeclampsia

Eclampsia

Abortion

Expulsion from the uterus of an embryo or fetus before the stage of viability (20 weeks' gestation or fetal weight less than 500 g). A distinction is made between abortion and premature birth: premature infants are those born after the stage of viability but before 37 weeks' gestation. Abortion may be either spontaneous (occurring from natural causes) or induced (artificial or therapeutic).

The second stage of labor is the longest stage. a. True b. False

False The 1st stage is longest

A 16-year-old primigravida, presents to the maternity clinic complaining of continual nausea and vomiting for the past 3 days. She states she is approximately 15 weeks pregnant and has been unable to hold anything down or take any fluids in without throwing up for the past 3 days. She reports she is dizzy and weak. On examination, Suzanne appears pale and anxious. Her mucous membranes are dry, skin turgor is poor, and her lips are dry and cracked. a. What is your impression of this condition? b. What risk factors does Suzanne have? c. What intervention is appropriate for this woman?

From her history, it appears she has hyperemesis gravidarum, because she is beyond the morning sickness time frame (6 to 12 weeks) and her symptoms are continual. Her risk factors include young age and primigravida status. Question Suzanne further concerning previous eating patterns and food intake. Ask what measures she has used at home to stop the nausea and vomiting. iConsult the HCP concerning hospitalization of Suzanne for IV therapy to correct hypovolemia and electrolyte imbalances. If home care is in order, advise her to avoid the intake of greasy or highly seasoned foods and to separate food from fluid intake; instruct her on antiemetic medication ordered and possible side effects; and instruct her to return to the clinic if symptoms do not subside within 48 hours.

An obese 39-year-old primigravida of African-American descent who is diagnosed with gestational hypertension presents to the prenatal clinic. Her history reveals that her sister developed preeclampsia during her pregnancy. When describing her diet to the nurse, this client mentions that she tends to eat a lot of fast food. a. What risk factors does this client have that increase her risk for gestational hypertension? b. When assessing this client, what assessment findings would lead the nurse to suspect that this client has developed severe preeclampsia?

Gloria is a primigravida with a family history of preeclampsia (her sister). She also is an obese woman who is older than 35 years of age. Her diet of fast foods is most likely inadequate in nutritional content. She is also African American. With severe preeclampsia, blood pressure is higher than 160/110 mm Hg on two occasions at least 6 hours apart, proteinuria is greater than 500 mg in 24 hours, and oliguria (less than 500 mL in 24 hours) is present. Other assessment findings may include pulmonary edema, cerebral or visual disturbances (altered level of consciousness, headache, blurred vision, and scotomata), hyperreflexia, and epigastric or right upper quadrant abdominal pain. Laboratory test findings would include increased hematocrit, creatinine, and uric acid levels, thrombocytopenia, and elevated liver enzymes.

*Complete abortion* (passage of all products of conception)

History of vaginal bleeding and abdominal pain *Passage of all fetal tissue* with subsequent decrease in pain and significant decrease in vaginal bleeding Mild uterine cramping Closed cervical opening Ultrasound demonstrating an empty uterus - mgmt: *No medical or surgical intervention necessary, Follow-up appointment to discuss family planning*

Ask a community health maternity nurse how the signs and symptoms of gestational hypertension (including preeclampsia and eclampsia) are taught, and how effective efforts have been to reduce the incidence in the area.

Hopefully the signs and symptoms would be taught to women during their 1st trimester, and written material would be handed out too. During each prenatal visit, the information should be reinforced to make sure women understand what they are and what to do about them if they should occur.

*Incomplete abortion* (passage of some of the products of conception)

Intense abdominal cramping *Heavy vaginal bleeding* Cervical dilation Open cervical os Ultrasound confirmation that products of conception still in uterus - mgmt: *Client stabilization, Evacuation of uterus via D&C or prostaglandin analog*

Antepartum Nurse

provides care to patients who have complications of pregnancy requiring hospitalization.

Select a childbirth website for expectant parents and critique the information provided in terms of its educational level and amount of advertising.

Many childbirth websites present very basic information about childbirth and attempt to target a wide audience of educational levels. Many of these sites promote various pregnancy and infant products.

*abruptio placentae*

Premature detachment of a normally situated placenta.

*abortion*

Nearly half of pregnancies among American women are unintended, and about 4 in 10 of these are terminated by abortion. Twenty-two percent of all pregnancies (excluding miscarriages) end in abortion - In September 2000, the U.S. Food and Drug Administration approved mifepristone to be marketed in the United States as an alternative to surgical abortion. - Medication abortion using mifepristone or a similar medication accounted for 38% of all nonhospital abortions, and about one quarter of abortions before 9 weeks' gestation - Abortion has become a hotly debated political issue that separates people into two camps: pro-choice and pro-life. The pro-choice group supports the right of any woman to make decisions about her reproductive functions based on her own moral and ethical beliefs. The pro-life group feels strongly that abortion is murder and deprives the fetus of the basic right to life. Both sides will continue to debate this very emotional issue for years to come. Medical and surgical modalities are available to terminate a pregnancy, depending on how far the pregnancy has developed. A surgical intervention can be performed up to 12 weeks' gestation; a medical intervention can be performed up to 9 weeks' gestation (King et al., 2015). All women undergoing abortion need emotional support, a stable environment in which to recover, and nonjudgmental care throughout the experience. Abortion is a complex issue, and the controversy is not only in the public arena; many nurses struggle with the conflict between their personal convictions and their professional duty. Nurses are taught to be supportive client advocates and to interact with a nonjudgmental attitude under all circumstances even when personal and political views differ from those of their clients. With all the advances in abortion care, this points toward greater nursing involvement. Although this bodes well for woman-centered care, the burden on nurses is likely to increase incrementally. This may have an adverse effect on the affective attributes or emotions that those nurses possess - Nurses need to clarify their personal values and beliefs on this issue and must be able to provide nonbiased care before assuming responsibility for clients who might be in a position to consider abortion. Their decision to care for or refuse to care for such clients affects staff unity, influences staffing decisions, and challenges the ethical concept of duty (McLemore, Kools, & Levi, 2015). The American Nurses Association's updated Code of Ethics for Nurses upholds the nurse's right to refuse to care for a client undergoing an abortion if the nurse ethically opposes the procedure (ANA, 2016). Nurses need to make their values and beliefs known to their managers before the situation occurs so that alternative staffing arrangements can be made. Open communication and acceptance of the personal beliefs of others can promote a comfortable working environment.

*Eclampsia*

Occurrence of one or more convulsions, not attributable to other cerebral conditions such as epilepsy or cerebral hemorrhage, in a patient with preeclampsia.

*The Baby Doe regulations*

an amendment to the US Child Abuse Protection and Treatment Act, provide specific guidelines on how to treat extremely ill, premature, terminally ill, and/or disabled infants regardless of the parents' wishes

*parens patriae*

Refusal of medical care may be considered a form of child neglect. If providing medical treatment may prevent substantial harm and suffering, or save a child's life, health care providers and the judicial system strive to advocate for the child. - The state has this overriding interest in the health and welfare of the child and can order that medical treatment to proceed without signed informed consent; the state has a right and a duty to protect children - if the parents refuse treatment and the health care team feels the treatment is reasonable and warranted, the case should be referred to the institution's ethics committee. - If the issue remains unresolved, or in complex cases, the judicial system may become involved

Teaching Guidelines 19.2 TEACHING FOR THE WOMAN WITH MILD PREECLAMPSIA

Rest in a quiet environment to prevent cerebral disturbances. Drink 8 to 10 glasses of water daily. Consume a balanced, high-protein diet including high-fiber foods. Obtain intermittent bed rest to improve circulation to the heart and uterus. Limit your physical activity to promote urination and subsequent decrease in blood pressure. Enlist the aid of your family so that you can obtain appropriate rest time. Perform self-monitoring as instructed, including: Taking your own blood pressure twice daily Checking and recording weight daily Performing urine dipstick twice daily Recording the number of fetal kicks daily Contact the home health nurse if any of the following occurs: Increase in blood pressure Protein present in urine Gain of more than 1 pound in 1 week Burning or frequency when urinating Decrease in fetal activity or movement Headache (forehead or posterior neck region) Dizziness or visual disturbances Increase in swelling in hands, feet, legs, and face Stomach pain, excessive heartburn, or epigastric pain Decreased or infrequent urination Contractions or low back pain Easy or excessive bruising Sudden onset of abdominal pain Nausea and vomiting

*passenger*

Size of the fetal head - has a *major* effect on the birth process. The sutures and fontanels make the skull flexible to accommodate during birth. The fetus (with placenta) is the passenger. The fetal head (size and presence of molding), fetal attitude (degree of body flexion), fetal lie (relationship of body parts), fetal presentation (first body part), fetal position (relationship to maternal pelvis), fetal station, and fetal engagement are all important factors that have an impact on the ultimate outcome in the birthing process. Take Note! *Parents may become concerned about the distortion of their newborn's head. However, reassurance that the oblong shape is only temporary is usually all that is needed to reduce their anxiety.*

*mgmt of eclampsia*

Severe preeclampsia may develop suddenly and bring with it high blood pressure of more than 160/110 mm Hg, proteinuria of more than 5 g in 24 hours, oliguria of less than 400 mL in 24 hours, cerebral and visual symptoms, and rapid weight gain. This clinical picture signals severe preeclampsia, and immediate hospitalization is needed. Treatment is highly individualized and based on disease severity and fetal age. Birth of the infant is the only cure, because preeclampsia depends on the presence of trophoblastic tissue. Therefore, the exact age of the fetus is assessed to determine viability. Severe preeclampsia is treated aggressively because hypertension poses a serious threat to mother and fetus. The goal of care is to stabilize the mother-fetus dyad and prepare for birth. Therapy focuses on controlling hypertension, preventing seizures, preventing long-term morbidity, and preventing maternal, fetal, or newborn death. Intense maternal and fetal surveillance starts when the mother enters the hospital and continues throughout her stay. The woman in labor with severe preeclampsia typically receives oxytocin to stimulate uterine contractions and magnesium sulfate to prevent seizure activity. Oxytocin and magnesium sulfate can be given simultaneously via infusion pumps to ensure both are administered at the prescribed rate. Magnesium sulfate is given intravenously via an infusion pump. A loading dose of 4 to 6 g is given over 5 minutes. Then, a maintenance dose of 2 g/hr is given. The client is evaluated closely for magnesium toxicity. If at all possible, a vaginal delivery is preferable to a cesarean birth for better maternal outcomes and less risk associated with a surgical birth. PGE2 gel may be used to ripen the cervix. A cesarean birth may be performed if the client is seriously ill. A pediatrician/neonatologist or neonatal nurse practitioner should be available in the birthing room to care for the newborn. A newborn whose mother received high doses of magnesium sulfate needs to be monitored for respiratory depression, hypocalcemia, and hypotonia. Decreased fetal heart rate variability may occur but, in general, magnesium sulfate does not pose a risk to the fetus. The newborn may exhibit respiratory depression, loss of reflexes, muscle weakness, and neurologic depression - As with any seizure, the initial management is to clear the airway and administer adequate oxygen. Positioning the woman on her left side and protecting her from injury during the seizure are key. Suction equipment must be readily available to remove secretions from her mouth after the seizure is over. IV fluids are administered after the seizure at a rate to replace urine output and additional insensible losses. Fetal heart rate is monitored closely. Magnesium sulfate is administered intravenously to prevent further seizures. Serum magnesium levels, respiratory rate, reflexes, and urine output in women receiving magnesium sulfate are closely monitored to avoid magnesium toxicity and prevent cardiac arrest. Calcium gluconate (1 g intravenously) is typically ordered to counteract magnesium toxicity. Hypertension is controlled with antihypertensive medications. After the woman's seizures are controlled, her stability is assessed. If she is found stable, birth via induction or cesarean birth is performed. If the woman's condition remains stable, she will be transferred to the postpartum unit for care. If she becomes unstable after giving birth, she may be transferred to the critical care unit for closer observation.

*passageway*

Soft Tissues of passageway: 1. Lower uterine segment 2. Cervix: *effaces (thins) and dilates (opens)* 3. Pelvic floor - muscular layer that separates the pelvic cavity above from the perineal space below 4. Vagina - stretches to accommodate the fetus 5. Introitus (external opening to the vagina) Take Note! *The process of cervical effacement and dilation is similar to that of pulling a turtleneck sweater over your head.*

Methotrexate (Trexall)

Stops growth of actively proliferating tissue - including fetal tissue Indicated for ectopic pregnancy, rheumatic conditions, psoriasis and chemotherapy May cause thrombocytopenia & other blood related disorders, neurotoxicity, N&V, fever, dizziness, diarrhea and pruritus Should not take meds stronger than analgesics such as acetaminophen Could mask symptoms of rupture Avoid alcohol, folic acid Reduce risk factors such as sexual intercourse with multiple partners or intercourse without a condom. Avoid contracting STIs that lead to PID. Obtain early diagnosis and adequate treatment of STIs. If an intrauterine contraceptive system is chosen, descriptions of the signs of PID should be included to reduce the risk of repeat ascending infections, which can be responsible for tubal scarring. Avoid smoking during childbearing years since a correlation and increase in risk exists. Use condoms to decrease the risk of infections that cause tubal scarring. Seek prenatal care early to confirm the location of pregnancy.

*surgical intervention for ectopic pregnancy*

Surgical management for the unruptured fallopian tube might involve a linear salpingostomy to preserve the tube—an important consideration for the woman wanting to preserve her future fertility. It may also be considered when medical treatment is considered unsuitable. With a ruptured ectopic pregnancy, surgery is necessary as a result of possible uncontrolled hemorrhage. A laparotomy with a removal of the tube (salpingectomy) may be necessary. With earlier diagnosis and medical management, the focus has changed from preventing maternal death to facilitating rapid recovery and preserving fertility. Regardless of the treatment approach (medical or surgical), the woman's beta-hCG level is monitored until it is undetectable to ensure that any residual trophoblastic tissue that forms the placenta is gone. Also, all Rh-negative unsensitized clients are given Rh immunoglobulin to prevent isoimmunization in future pregnancies.

During clinical post-conference, share with the other nursing students how the critical forces of labor influenced the length of labor and the birthing process for a laboring woman assigned to you.

This discussion should involve the passenger, powers, passageway, position, and psychological response of the student's assigned women going through labor and how each affected the length and stages of labor.

placenta accrete

The abnormal adherence of the chorionic villi to the myometrium, associated with partial or complete absence of the decidua basalis and, in particular, the stratum spongiosum.

ANTHROPOID PELVIS

The anthropoid pelvis is common in men and is most common in non-White women - It occurs in approximately 25% of women - The pelvic inlet is oval and the sacrum is long, producing a deep pelvis (wider front to back [anterior to posterior] than side to side [transverse]). - caginal birth is more favorable with this pelvic shape compared with the android or platypelloid shape

molding

The changed (elongated) shape of the fetal skull at birth as a result of overlapping of the cranial bones

A nurse is caring for a client administered general anesthesia for an emergency cesarean birth. The nurse notes the client's uterus is relaxed upon massage. What would the nurse do next? a. Continue to monitor the client. b. Continue to massage the client's fundus. c. Administer oxygen to the client. d. Assess the client's vaginal bleeding.

b. Continue to massage the client's fundus.

A nurse is monitoring a fetal heart rate (FHR) pattern on her client in labor. The earlier baseline FHR was 140. The FHR now is 168. *The nurse knows that which factors can affect changes in the FHR?* Select all that apply. a. narcotic medication to maternal client b. fetal movement c. fetal distress d. utero-placental insufficiency e. maternal fever

b. fetal movement c. fetal distress e. maternal fever

Compare and contrast a local birthing center to a community hospital's birthing suite in terms of the pain management techniques and fetal monitoring used.

The findings will vary from facility to facility, but the student might find a more liberal use of nonpharmacologic techniques in the birthing center compared to the hospital setting and more frequent use of hydrotherapy and ambulation to relieve discomfort. Also, intermittent assessment using a hand-held Doppler is probably used more frequently in the birthing center compared to the hospital, where continuous electronic fetal monitoring is prevalent.

*treatment for molar pregnancy*

The nurse plays a crucial role in identifying and bringing this condition to the attention of the health care provider based on sound knowledge of the typical clinical manifestations and through astute antepartal assessments. Clinical manifestations of GTD are very similar to those of spontaneous abortion at about 12 weeks of pregnancy. Assess the woman for potential clinical manifestations at each antepartal visit. Be alert for the following: Report of early signs of pregnancy, such as amenorrhea, breast tenderness, fatigue Brownish vaginal bleeding/spotting Anemia Inability to detect a fetal heart rate after 10 to 12 weeks' gestation Fetal parts not evident with palpation Bilateral ovarian enlargement caused by cysts and elevated levels of hCG Persistent, often severe, nausea and vomiting (due to high hCG levels) Fluid retention and swelling Uterine size larger than expected for pregnancy dates Extremely high hCG levels present; no single value considered diagnostic Early development of preeclampsia (usually not present until after 24 weeks) Absence of fetal heart rate or fetal activity Expulsion of grape-like vesicles (possible in some women) The diagnosis is made by very high hCG levels and the characteristic appearance of the vesicular molar pattern in the uterus via transvaginal ultrasound. - Nursing management of the woman with GTD focuses on preparing her for a D&C, providing emotional support to deal with the loss and potential risks, and educating her about the risk that cancer may develop after a molar pregnancy and the strict adherence needed with the follow-up program. The woman must understand the need for the continued follow-up care regimen to improve her chances of future pregnancies and to ensure her continued quality of life.

*intervening for eclampsia*

The onset of seizure activity identifies eclampsia. Typically, eclamptic seizures are generalized and start with facial twitching. The body then becomes rigid, in a state of tonic muscular contraction. The clonic phase of the seizure involves alternating contraction and relaxation of all body muscles. Respirations stop during seizure activity and resume shortly after it ends. Client safety is the primary concern during eclamptic seizures. If possible, turn the client to her side and remain with her. Make sure that the side rails are up and padded. Dim the lights and keep the room quiet. Document the time and sequence of events as soon as possible. After the seizure activity has ceased, suction the nasopharynx as necessary and administer oxygen. Continue the magnesium sulfate infusion to prevent further seizures. Ensure continuous electronic fetal monitoring, evaluating fetal status for changes. Also assess the client for uterine contractions. After the client is stabilized, prepare her for the birthing process as soon as possible to reduce the risk of perinatal mortality.

Interview a woman on the mother-baby unit who has given birth within the past few hours. Ask her to describe her experience and examine psychological factors that may have influenced her laboring process.

This discussion will vary depending on the women's labor and birth experience. *Psychological factors that could be addressed might include previous birth experiences, age, pregnancy discomforts, cultural beliefs, expectations for this birth experience, preparation for birth, and effectiveness and participation of support system!!!*

*mgmt of HELLP*

The treatment for HELLP syndrome is based on the severity of the disease, the gestational age of the fetus, and the condition of the mother and fetus. The mainstay of treatment is lowering of high blood pressure with rapid-acting antihypertensive agents, prevention of convulsions or further seizures with magnesium sulfate, and use of steroids for fetal lung maturity if necessary, followed by the birth of the infant and placenta (Foley et al., 2014). The client should be admitted or transferred to a tertiary center with a neonatal intensive care unit. Additional treatment includes correction of the coagulopathies that accompany HELLP syndrome. After this syndrome is diagnosed and the woman's condition is stable, birth of the infant is indicated. Magnesium sulfate is used prophylactically to prevent seizures. Antihypertensives such as hydralazine or labetalol are given to control blood pressure. Blood component therapy - such as fresh-frozen plasma, packed red blood cells, or platelets - is transfused to address the microangiopathic hemolytic anemia. Birth may be delayed up to 96 hours so that betamethasone or dexamethasone can be given to stimulate lung maturation in the preterm fetus. - Nursing management of the woman diagnosed with HELLP syndrome is the same as that for the woman with severe preeclampsia. If possible, the woman with HELLP syndrome should be transferred to a tertiary care center once she has been assessed and stabilized. Closely monitor the client for changes and provide ongoing support throughout this experience.

*intervening for severe preeclampsia*

The woman with severe preeclampsia requires hospitalization. Maintain the client on complete bed rest in the left lateral lying position. Ensure that the room is dark and quiet to reduce stimulation. Give sedatives as ordered to encourage quiet bed rest. The client is at risk for seizures if the condition progresses. Therefore, institute and maintain seizure precautions, such as padding the side rails and having oxygen, suction equipment, and call light readily available to protect the client from injury. - Closely monitor the client's blood pressure. Administer antihypertensives as ordered to reduce blood pressure (Drug Guide 19.3). Assess the client's vision and level of consciousness. Report any changes and any complaints of headache or visual disturbances. Offer a high-protein diet with 8 to 10 glasses of water daily. Monitor the client's intake and output every hour and administer fluid and electrolyte replacements as ordered. Assess the woman for signs and symptoms of pulmonary edema, such as crackles and wheezing heard on auscultation, dyspnea, decreased oxygen saturation levels, cough, neck vein distention, anxiety, and restlessness. The treatment of acute pulmonary edema is symptomatic and includes the administration of vasodilating agents and of diuretics. The development of acute pulmonary edema in women with hypertension during pregnancy is associated with high levels of IV fluid administration - *Take Note! Preeclampsia increases the risk of placental abruption, preterm birth, intrauterine growth restriction, and fetal distress during childbirth. Always be prepared if you see symptoms of preeclampsia!* - To achieve a safe outcome for the fetus, prepare the woman for possible testing to evaluate fetal status as preeclampsia progresses. Testing may include the nonstress test, serial ultrasounds to track fetal growth, amniocentesis to determine fetal lung maturity, Doppler velocimetry to screen for fetal compromise, and biophysical profile to evaluate ongoing fetal well-being. Other laboratory tests may be performed to monitor the disease process and to determine if it is progressing into HELLP syndrome. These include liver enzymes such as lactic dehydrogenase (LDH), ALT, and AST; chemistry panel, such as creatinine, BUN, uric acid, and glucose; CBC, including platelet count; coagulation studies, such as PT, PTT, fibrinogen, and bleeding time; and a 24-hour urine collection for protein and creatinine clearance. Administer parenteral magnesium sulfate as ordered to prevent seizures. Assess DTRs to evaluate the effectiveness of therapy. Clients with preeclampsia commonly present with hyperreflexia. Severe preeclampsia causes changes in the cortex, which disrupts the equilibrium of impulses between the cerebral cortex and the spinal cord. Brisk reflexes (hyperreflexia) are the result of an irritable cortex and indicate central nervous system (CNS) involvement. Diminished or absent reflexes occur when the client develops magnesium toxicity. Because magnesium is a potent neuromuscular blockade, the afferent and efferent nerve pathways do not relay messages properly and hyporeflexia develops. Common sites used to assess DTRs are biceps reflex, triceps reflex, patellar reflex, Achilles reflex, and plantar reflex. Nursing Procedure 19.1 highlights the steps for assessing the patellar reflex. The National Institute of Neurological Disorders and Stroke, a division of the National Institutes of Health, published a scale in the early 1990s that, although subjective, is used widely today. It grades reflexes from 0 to 4+. Grades 2+ and 3+ are considered normal, whereas grades 0 and 4 may indicate pathology (Table 19.2). Because these are subjective assessments, to improve communication of reflex results, condensed descriptor categories such as absent, average, brisk, or clonus should be used rather than numeric codes. Clonus is the presence of rhythmic involuntary contractions, most often at the foot or ankle. Sustained clonus confirms CNS involvement. Nursing Procedure 19.2 highlights the steps when testing for ankle clonus. With magnesium sulfate administration, the client is at risk for magnesium toxicity. Closely assess the client for signs of toxicity, which include a respiratory rate of less than 12 breaths per minute, absence of DTRs, and a decrease in urinary output (<30 mL/hr). Also monitor serum magnesium levels. Although exact levels may vary among agencies, serum magnesium levels ranging from 4 to 7 mEq/L are considered therapeutic, whereas levels more than 8 mEq/dL are generally considered toxic. As levels increase, the woman is at risk for severe problems: 10 mEq/L: possible loss of DTRs 15 mEq/L: possible respiratory depression 25 mEq/L: possible cardiac arrest - If signs and symptoms of magnesium toxicity develop, expect to administer calcium gluconate as the antidote. Throughout the client's stay, closely monitor her for signs and symptoms of labor. Perform continuous electronic fetal monitoring to assess fetal well-being. Note trends in baseline rate and presence or absence of accelerations or decelerations. Also observe for signs of fetal distress and report them immediately. Administer glucocorticoid treatment as ordered to enhance fetal lung maturity and prepare for labor induction if the mother's condition warrants. - Keep the client and family informed of the woman's condition and educate them about the course of treatment. Provide emotional support for the client and family. Severe preeclampsia is very frightening for the client and her family, and most expectant mothers are very anxious about their own health as well as that of the fetus. To allay anxiety, use light touch to comfort and reassure her that the necessary actions are being taken. Actively listening to her concerns and fears and communicating them to the health care provider are important to keep open the lines of communication. Offering praise for small accomplishments can provide positive reinforcement for effective behaviors.

What manifestations would be found if Cindy is experiencing true labor?

There would be progressive dilation and effacement of her cervix if true labor is occurring. Contraction pain also would not be relieved with walking, and the pain would start in the back and radiate around toward the front of the abdomen. Contractions also would occur regularly, becoming closer together, usually 4 to 6 minutes apart, and last 30 to 60 seconds. If she is experiencing false labor, slight effacement might be present, but not dilation.

Share experiences within a post clinical conference group regarding the pain management interventions of the clients to which you were assigned. Compare and evaluate the effectiveness of different methods used, maternal behavior observed, and neonatal outcome in terms of Apgar scores.

This information will vary depending on what the woman reports to the student. Unrealistic pain management plans need to be identified and valid evidence-based ones presented to the woman. Misconceptions can be cleared up also.

Cephalic presentation refers to a fetus whose head enters the pelvic inlet first. a. True b. False

True

There are four essential components of labor. The first is the passageway. It is composed of the bony pelvis and soft tissues. What is one component of the passageway? a. false pelvis b. cervix c. perineum d. uterus

b. cervix

*umbilical cord blood banking*

Umbilical cord blood, the remaining blood in the umbilical cord at birth, can be collected and be a source of stem cells for an individual in need of a bone marrow transplant later in life. It can also be used for admission laboratory studies in neonates and also for a transfusion if needed - The extraordinary scientific and technologic advance of contemporary medicine constantly leads toward the introduction of new treatments. Umbilical cord blood is a potential vast source of primitive hematopoietic stem and progenitor cells available for clinical application. Cord blood can be used as an alternative source for bone marrow transplantation and its use is developing into a new field of treatment for pediatric and adult clients presenting with hematologic disorders, immunologic defects, and specific genetic diseases (Roura et al., 2015). Blood from a newborn's umbilical cord, once considered a waste product that was routinely discarded along with the placenta, is now considered to contain potentially life-saving stem cells. Banks were initially developed to store cord blood stem cells from newborns, for a fee, for potential future use by the same child or a family member if the child developed disease later in life. Today, there are public banks that store, for free, stem cells that can be used by anyone needing them, similar to how public blood banks work. Pregnant women should be aware that stem cells from cord blood cannot currently be used to treat inborn errors of metabolism or other genetic diseases in the same individual from which they were collected because the cord blood would have the same genetic mutation. Cord blood collected from a newborn that later develops childhood leukemia cannot be used to treat that leukemia for much the same reason. Umbilical cord banks are a central component, as umbilical cord tissue providers, in both medical treatment and scientific research with stem cells. But, whereas the creation of umbilical cord banks is seen as successful practice, others perceive it as ethically risky. The fact that private cord banks offer their services as "biologic insurance" in order to obtain informed consent by promising the parents that the tissue that will be stored to insure the health of their child in the future raises the issue of whether the consent is freely given or given under coercion. Another consideration that must be made in relation to privately owned cord banks involves the ownership of the stored umbilical cord. Conflicts between moral principles and economic interests (many physicians own private blood banks) cause dilemmas in the clinical practice of umbilical cord blood storage and use, especially in privately owned banks (Ballen, Verter, & Kurtzberg, 2015). Both the ACOG (2012) and AAP (2012) have reaffirmed previously issued statements opposing the use of for-profit banks and criticizing their marketing tactics. Instead, they recommended that parents donate cord blood to public banks, which make it available for free to anyone who needs it. Globally, other organizations have done the same. Private umbilical cord blood banking raises a question of special legal regulation. This practice promises the safe storage of biologic material on the assumption that it may be useful, at a certain moment in future, for its own donor (or for a donor's close family member) for curing serious blood diseases (Kim, Han, & Shin, 2015). Nurses need to provide unbiased education about this topic with all pregnant women and their families to allow informed choices to be made.

Discuss various activities a woman with a multiple gestation could engage in to help pass the time when ordered to be on bed rest at home for 2 months

Various activities for the woman on prolonged bed rest at home could include watching TV, reading, visiting computer sites with chat rooms, talking on the telephone, playing cards or engaging in crafts, having visitors in frequently, and completing educational courses online. The woman could also use the time to develop lists for managing the house while on bed rest, read or play games with her other children, and expand her knowledge related to the upcoming birth of her babies.

A new OB/GYN care provider has just finished evaluating her one hundredth client. In reviewing the documentation from all clients thus far, which types of pelvis would the nurse assume the care provider has seen the most and the least? a. gynecoid and android, respectively b. gynecoid and platypelloid, respectively c. anthropoid and gynecoid, respectively d. android and platypelloid, respectively

b. gynecoid and platypelloid, respectively

A nurse recommends to a client in labor to try concentrating intently on a photo of her family as a means of managing pain. The woman looks skeptical and asks, "How would that stop my pain?" Which explanation should the nurse give? a. "It distracts your brain from the sensations of pain." b. "It causes the release of endorphins." c. "It blocks the transmission of nerve messages of pain at the receptors." d. "It disrupts the nerve signal of pain via mechanical irritation of the nerves."

a. "It distracts your brain from the sensations of pain."

A woman is documented on the labor and birth board to be 7 cm dilated. Her family wants to know how long she will be in labor. The nurse should provide which information to the family? a. "She is in active labor; she is progressing at this point and we will keep you posted." b. "She is in the transition phase of labor, and it will be with in 2 to 3 hours, though it might be sooner." c. "She is still in early latent labor and has much too long to go to tell when she will give birth." d. "She is doing well and is in the second stage; it could be anytime now."

a. "She is in active labor; she is progressing at this point and we will keep you posted."

A client, 38 weeks gestation and pregnant with her first child, calls the clinic and states, "My baby is lower, and it is more difficult to walk." She asks if she should come to the hospital to be checked. How should the nurse respond? a. "The baby has dropped into the pelvis; your body and baby are getting ready for labor in the next few weeks." b. "This is not normal unless you are in active labor; come to the hospital and be checked." c. "That is something we expect with a second or third baby, but because it is your first, you need to be checked." d. "The baby moved down into the pelvis; this means you will be in labor within 24 hours, so wait for contractions to come to the hospital."

a. "The baby has dropped into the pelvis; your body and baby are getting ready for labor in the next few weeks."

A nurse is performing a vaginal examination of a woman in the early stages of labor. The woman has been at 2 cm dilated for the past 2 hours, but effacement has progressed steadily. Which statement by the nurse would best encourage the client regarding her progress? a. "You are still 2 cm dilated, but the cervix is thinning out nicely." b. "There has been no further dilatation; effacement is progressing." c. "You haven't dilated any further, but hang in there; it will happen eventually." d. Don't mention anything to the client yet; wait for further dilatation to occur.

a. "You are still 2 cm dilated, but the cervix is thinning out nicely."

When caring for a client during the active phase of labor without continuous electronic fetal monitoring, the nurse would intermittently assess FHR every: a. 15 to 30 minutes b. 5 to 10 minutes c. 45 to 60 minutes d. 60 to 75 minutes

a. 15 to 30 minutes Several professional women's health organizations have published guidelines concerning the timing of intermittent FHR assessments during the active stage of labor. The current recommendation is that intermittent FHR is assessed every 15 minutes during the active phase of labor.

The nurse is caring for a client in active labor who has had a fetal blood sampling to check for fetal hypoxia. The nurse determines that the fetus has acidosis when the pH is: a. 7.15 or less. b. 7.25 or more. c. 7.20. d. 7.21.

a. 7.15 or less.

A client has just received IV sedation. What must the nurse tell the client to do? a. Ambulate only with assistance from the nurse or caregiver. b. Ambulate within 15 minutes to prevent spinal headache. c. Sit on the edge of the bed with her feet dangling before ambulating. d. Remain in bed for at least 30 minutes.

a. Ambulate only with assistance from the nurse or caregiver.

Which statement is true regarding analgesia versus anesthesia? a. Analgesia only reduces pain, but anesthesia partially or totally blocks all pain in a particular area. b. Decreased FHR variability is a common side effect when regional anesthesia is used. c. Regional anesthesia should be given with caution close to the time of birth because it crosses the placenta and can cause respiratory depression in the newborn. d. Hypotension is the most common side effect when systemic analgesia is used.

a. Analgesia only reduces pain, but anesthesia partially or totally blocks all pain in a particular area.

A primigravida at 10 weeks reports slight vaginal spotting without passage of tissue and mild uterine cramping. When examined, no cervical dilation is noted. The nurse caring for this woman would: a. Anticipate she will be sent home and placed on bed rest with instructions to avoid stress and intercourse b. Prepare the woman for a dilation and curettage c. Notify a grief counselor to assist the woman with the imminent loss of her fetus d. Prepare for the doctor to perform a cerclage to help her maintain her pregnancy

a. Anticipate she will be sent home and placed on bed rest with instructions to avoid stress and intercourse the woman is experiencing a threatened abortion therefore, a conservative approach is attempted 1st c reflects management of an inevitable and complete or incomplete abortion b. blood tests for HCG and progesterone levels would be done d. cerclage or suturing of the cervix is done for recurrent, spontaneous abortion associated with premature dilation of the cervix

A nurse caring for a pregnant client in labor observes that the fetal heart rate (FHR) is below 110 beats per minute. Which interventions should the nurse perform? Select all that apply.

a. Assess client for underlying causes. b. Turn the client on her left side. c. Administer oxygen by mask.

During the second stage of labor, assessment would include which of the following? a. Complaints of rectal or perineal pressure b. Estimated date of birth c. Fundal height d. Fetal position

a. Complaints of rectal or perineal pressure Pushing now Important during 2nd stage The rest are important for 1st stage

7. A woman is being discharged after receiving treatment for a hydatidiform molar pregnancy. The nurse should include which of the following in her discharge teaching? a. Do not become pregnant for at least a year; use contraceptives to prevent it b. Have the client's blood pressure checked weekly in the clinic c. RhoGAM must be given within the next month to her at the clinic d. An amniocentesis can detect a recurrence of this disorder in the future

a. Do not become pregnant for at least a year; use contraceptives to prevent it Assessment of serum chorionic gonadotropin (hCG) is considered a specific tumor marker for gestational trophoblastic disease that isn't resolved. hCG levels are assayed at frequent intervals for up to a year. Pregnancy would obscure the evidence of choriocarcinoma by the normal secretion of hCG. Response "B" is incorrect because there is not a direct link between a molar pregnancy and hypertension continuing after the uterus is empty. Response "C" is incorrect because RhoGAM is given for Rh incompatibility to prevent sensitivity, not as treatment for a molar pregnancy. Response "D" is incorrect because an amniocentesis is not diagnostic of a molar pregnancy in the future.

Which of the following practices would not be included in a physiologic birth? a. Early induction of labor <39 weeks' gestation b. Freedom of movement for the laboring woman c. Continuous presence and support throughout labor d. Encouraging spontaneous pushing when urge felt

a. Early induction of labor <39 weeks' gestation since inducing labor artificially, rather than waiting for spontaneous labor to start doesn't provide for a physiologic birth. Nature should be allowed to take its course without artificial means to initiate labor. Responses "B," "C," and "D" all contribute to physiologic birth practices.

13. The cardinal movements of labor include which of the following? Select all that apply. a. Extension and rotation b. Descent and engagement c. Presentation and position d. Attitude and lie e. Flexion and expulsion

a. Extension and rotation b. Descent and engagement e. Flexion and expulsion The cardinal movements of labor by the fetus include: engagement descent flexion international rotation extension external rotation expulsion The other choices describe the various fetal positions.

How does a woman who feels in control of the situation during labor influence her pain? a. Feelings of control are inversely related to the client's report of pain. b. Decreased feeling of control helps during the third stage. c. There is no association between the two factors. d. Feeling in control shortens the overall length of labor.

a. Feelings of control are inversely related to the client's report of pain.

Which of the following women should receive RhoGAM postpartum? a. Nonsensitized Rh-negative mother with a Rh-negative newborn b. Nonsensitized Rh-negative mother with a Rh-positive newborn c. Sensitized Rh-negative mother with a Rh-positive newborn d. Sensitized Rh-negative mother with a Rh-negative newborn

a. Nonsensitized Rh-negative mother with a Rh-negative newborn

When caring for a client in the third stage of labor, the nurse notices that the expulsion of the placenta has not occurred within 5 minutes after birth of the infant. What should the nurse do? a. Nothing. Normal time for stage three is 5 to 30 minutes. b. Notify the primary care provider of the problem. c. Increase the IV tocolytic to help in expulsion of the placenta. d. Do a vaginal exam to see if the placenta is stuck in the birth canal.

a. Nothing. Normal time for stage three is 5 to 30 minutes.

Interventions that are underutilized in promoting a normal birth. Select all that apply. a. Oral nutrition and fluids in labor b. Open glottis pushing in the second stage of labor c. Skin-to-skin contact after birth for infant bonding d. Routine artificial rupture of membranes (amniotomy) e. Labor induction with Pitocin given intravenously f. Routine episiotomy to shorten labor length

a. Oral nutrition and fluids in labor b. Open glottis pushing in the second stage of labor c. Skin-to-skin contact after birth for infant bonding since all of these are evidence-based interventions that are physiologically sound without placing the mother or the neonate in any danger. Food and clear fluids provide hydration and nutrition and give comfort to laboring women. Fasting during labor will increase gastric acid production. Open glottis while pushing allows the woman's body to sense the urge to push naturally. Skin-to-skin contact promotes mother-infant bonding and warmth. Incorrect responses would include "D," "E," and "F" since these are artificial means to speed up the labor process which places the mother and newborn in jeopardy. Amniotomy may be associated with umbilical cord prolapse and fetal heart rate decelerations. Episiotomy is associated with an increase in third- and fourth-degree perineal lacerations, discomfort, and healing delays. Induction with Pitocin may cause tetanic contractions causing hypoxia to the fetus.

A patient at 38 weeks diagnosed with marginal placenta previa has just given birth to a healthy newborn male. The nurse recognizes that the immediate focus for the care of this patient should be: a. Preventing hemorrhage b. Relieving pain c. Preventing infection d. Fostering attachment of the woman with her baby

a. Preventing hemorrhage hemorrhage is a major potential postpartum complication because the implantation site of the placenta is in the lower uterine segment, which has a limited capacity to contract after birth infection is another major complication, but it is not the immediate focus of care b and d are also important but not to the same degree as hemorrhage, which is life threatening.

A nurse is coaching a woman during the second stage of labor. Which action should the nurse encourage the client to do at this time? a. Push with contractions and rest between them. b. Hold the breath while pushing during contractions. c. Begin pushing as soon as the cervix has dilated to 8 cm. d. Pant while pushing.

a. Push with contractions and rest between them.

The nurse is reviewing the uterine contraction pattern and identifies the peak intensity, documenting this as which phase of the contraction? a. acme b. increment c. decrement d. diastole

a. acme

When determining the frequency of contractions, the nurse would measure which of the following? a. Start of one contraction to the start of the next contraction b. Beginning of one contraction to the end of the same contraction c. Peak of one contraction to the peak of the next contraction d. End of one contraction to the beginning of the next contraction

a. Start of one contraction to the start of the next contraction Frequency is measured from the start of one contraction to the start of the next contraction. The duration of a contraction is measured from the beginning of one contraction to the end of that same contraction. The intensity of two contractions is measured by comparing the peak of one contraction with the peak of the next contraction. The resting interval is measured from the end of one contraction to the beginning of the next contraction.

3. RhoGAM is given to Rh-negative women to prevent maternal sensitization. In addition to pregnancy, Rh-negative women would also receive this medication after which of the following? a. Therapeutic or spontaneous abortion b. Head injury from a car accident c. Blood transfusion after a hemorrhage d. Unsuccessful artificial insemination procedure

a. Therapeutic or spontaneous abortion Any time there is a pregnancy with the chance of maternal and fetal blood mixing, RhoGAM is needed to prevent sensitization or antibody production. Head injury resulting from a car crash is not a situation in which there would be mixing of fetal or maternal blood. The trauma would cause hemorrhage, but not a sensitization reaction. A blood transfusion after hemorrhage would require typing and cross-matching of the client's blood; thus, she would receive blood with her own Rh factor, not one with Rh-positive blood. Because the artificial insemination procedure was unsuccessful, no pregnancy occurred and RhoGAM would not be necessary.

Semi-Fowler's position. What is the appropriate rationale for this measure? a. To prevent supine hypotension syndrome b. To decrease the heart rate of the fetus c. To aid the woman as she pushes during labor d. To prevent the woman from falling out of bed

a. To prevent supine hypotension syndrome

When explaining to a class of pregnant women why labor begins, the nurse will include the fact that there are several theories that have been proposed to explain why labor begins, although none have been proven scientifically. Which idea is one of those theories? a. change in estrogen-to-progesterone ratio b. decrease in number of oxytocin receptors c. decrease in the level of estrogen d. decrease in prostaglandins, leading to myometrium contractions

a. change in estrogen-to-progesterone ratio

The nurse explains Leopold's maneuvers to a pregnant client. For which purposes are these maneuvers performed? Select all that apply. a. determining the presentation of the fetus b. determining the position of the fetus c. determining the lie of the fetus d. determining the weight of the fetus e. determining the size of the fetus

a. determining the presentation of the fetus b. determining the position of the fetus c. determining the lie of the fetus

*A nurse is reviewing the FHR and notes it to be in the range of 100 to 106 bpm over the past 10 minutes. Which conditions might the nurse suspect as the cause?* Select all that apply.

a. fetal hypoxia b. effect of maternal analgesia c. prolonged umbilical cord compression

Which type of pelvis has a roomy, round inlet and is most favorable for vaginal birth? a. gynecoid b. android c. anthropoid d. platypelloid

a. gynecoid

The nurse is teaching a prenatal class the signs and symptoms of true labor. What is the appropriate response by the class that shows an understanding of true labor contractions? a. increase even if I relax and take a shower b. remain irregular with the same intensity c. subside when I walk around and use the lateral position d. cause discomfort over the top of my uterus

a. increase even if I relax and take a shower

A nurse is caring for a pregnant client who is in labor. Which maternal physiologic responses should the nurse monitor for in the client as the client progresses through birth? Select all that apply.

a. increase in BP b. increase in HR c. increase in RR

A nurse is caring for a pregnant client with rhythmic uterine contractions. Which feature should the nurse identify as associated with true labor? a. increase in frequency of the contractions b. decrease in the intensity of the contractions c. irregularity in the duration of the contractions d. lessening of the contractions with position change

a. increase in frequency of the contractions

A nurse sees a pregnant client at the clinic. The client is close to her due date. During the visit the nurse would emphasize that the client get evaluated quickly should her membranes rupture spontaneously based on the understanding of which possibility? a. increased risk of infection b. potential rapid birth of fetus c. potential placenta previa d. increased risk of breech presentation

a. increased risk of infection

Place the following stages of labor in order from what occurs first to last. All options must be used. LAT23

a. latent stage b. active stage c. transition stage d. second stage e. third stage

The five "Ps" of labor are: a. passageway, passenger, position, powers, psych. b. passenger, posture, position, presentation, psych. c. passenger, position, presentation, pushing, psych. d. passenger, position, powers, presentation, psych.

a. passageway, passenger, position, powers, psych.

When going through the transition phase of labor, women often feel out of control. What do women in the transition phase of labor need the most? a. positive reinforcement b. their significant other beside them c. intense nursing care d. just to be left alone

a. positive reinforcement

A nurse is caring for a client who has been administered an epidural block. Which should the nurse assess next? a. respiratory rate b. temperature c. pulse d. uterine contractions

a. respiratory rate

A young couple are anxious for the birth of their child. This is the couple's first child. The nurse discusses the options, including elective induction of labor for nulliparas. The nurse determines that additional teaching is necessary when the couple identifies which condition as being associated with this procedure? a. shorter hospitalizations b. increased rates of cesarean sections c. postpartum hemorrhage d. neonatal resuscitation

a. shorter hospitalizations

To assess the frequency of a woman's labor contractions, the nurse would time: a. the beginning of one contraction to the beginning of the next. b. the end of one contraction to the beginning of the next. c. the interval between the acme of two consecutive contractions. d. how many contractions occur in 5 minutes.

a. the beginning of one contraction to the beginning of the next.

A 19-year-old female presents in advanced labor. Examination reveals the fetus is in frank breech position. The nurse interprets this finding as indicating: a. the buttocks are presenting first with both legs extended up toward the face. b. the fetus is sitting cross-legged above the cervix. c. one leg is presenting. d. one arm is presenting.

a. the buttocks are presenting first with both legs extended up toward the face.

A nurse is caring for a pregnant client at her 34-week checkup. The client has chosen the Lamaze method for her birthing plan but states that her partner does not agree. The client says she will just change her plan. Which response by the nurse would be appropriate to support the female client? a. "Just wait until the birth; your partner's mind could change." b. "Have you and your partner discussed what his/her role will be in the birth?" c. "Preparing for the birth works for some clients, but not for all." d. "Maybe you should choose a different support person."

b. "Have you and your partner discussed what his/her role will be in the birth?"

A woman is suspected of having abruptio placentae. Which of the following would the nurse expect to assess as a classic symptom? a. Painless, bright-red bleeding b. "Knife-like" abdominal pain c. Excessive nausea and vomiting d. Hypertension and headache

b. "Knife-like" abdominal pain When the placenta separates from the uterine wall, it causes irritation and bleeding into the muscle fibers, which causes pain Painless, bright-red bleeding indicates placenta previa symptomatology Excessive nausea and vomiting would be characteristic of hyperemesis gravidarum. Hypertension and headache would be associated with gestational hypertension.

A primigravida, who is 1 cm dilated, is in early latent labor. She has expressed a desire to avoid epidural anesthesia and asks about nonpharmacologic options for pain relief as her labor progresses. How could the nurse appropriately respond? Select all that apply. a. "If you are already asking about pain relief at this point in labor, you will most likely end up with an epidural anyway." b. "The tub usually is not recommended when you are at such an early stage because it can cause your labor to slow down. Let's talk about using the tub as your labor progresses." c. "I can show you some simple breathing exercises that can help you relax." d. "If you haven't already practiced these techniques, meditation and imagery won't work." e. "You may want to go for a walk now. When you come back, I'll show you how to use the birthing ball."

b. "The tub usually is not recommended when you are at such an early stage because it can cause your labor to slow down. Let's talk about using the tub as your labor progresses." c. "I can show you some simple breathing exercises that can help you relax." e. "You may want to go for a walk now. When you come back, I'll show you how to use the birthing ball."

When assessing fetal heart rate patterns, which finding would alert the nurse to a possible problem? a. variable decelerations b. prolonged decelerations c. early decelerations d. accelerations

b. *prolonged decelerations*

6. A laboring woman is admitted to the labor and birth suite at 6 cm dilation. She would be in which phase of the first stage of labor? a. Latent b. Active c. Transition d. Early

b. Active Cervical dilation of 6 cm indicates that the woman is in the active phase of the first stage of labor. In this phase, the cervix dilates from 3 to 7 cm with 40% to 80% effacement occurring. During the latent phase, the cervix dilates from 0 to 3 cm. During the transition phase, the cervix dilates from 8 to 10 cm. The first stage of labor is divided into three phases: latent, active, and transition. There is no early phase.

Braxton Hicks contractions are termed "practice contractions" and occur throughout pregnancy. When the woman's body is getting ready to go into labor, it begins to show anticipatory signs of impending labor. Among these signs are Braxton Hicks contractions that are more frequent and stronger in intensity. What differentiates Braxton Hicks contractions from true labor? a. Braxton Hicks contractions get closer together with activity. b. Braxton Hicks contractions usually decrease in intensity with walking. c. Braxton Hicks contractions do not last long enough to be true labor. d. Braxton Hicks contractions cause "ripening" of the cervix.

b. Braxton Hicks contractions usually decrease in intensity with walking.

A 34-year-old pregnant patient has had a consistently high BP ranging from 148/92 to 160/98 since she was 28 years old. Her weight gain has followed normal patterns, and urinalysis remains normal as well. a. Eclampsia b. Chronic hypertension c. Gestational hypertension d. Preeclampsia

b. Chronic hypertension

A client in her third trimester of pregnancy arrives at a health care facility with a report of cramping and low back pain; she also notes that she is urinating more frequently and that her breathing has become easier the past few days. Physical examination conducted by the nurse indicates that the client has edema of the lower extremities, along with an increase in vaginal discharge. What should the nurse do next? a. Notify the health care provider. b. Continue to monitor the client. c. Assess the client's blood pressure. d. Prepare the client for birth.

b. Continue to monitor the client.

When teaching a group of nursing students about the stages of labor, the nurse explains that softening, thinning, and shortening of the cervical canal occur during the first stage of labor. Which term is the nurse referring to in the explanation? a. Crowning b. Effacement c. Dilatation d. molding

b. Effacement

The primary expected outcome for care associated with the administration of magnesium sulfate would be met if the woman exhibits which of the following? a. Decrease in both systolic and diastolic BP b. Experiences no seizures c. States she feels more relaxed and calm d. Urinates more frequently, resulting in a decrease in pathologic edema

b. Experiences no seizures magnesium sulfate is a CNS depressant given to prevent seizures. Magnesium sulfate is a central nervous system depressant that interferes with calcium uptake in the cells of the myometrium, thus reducing the muscular ability to contract. Magnesium sulfate is not used as supplementation during pregnancy because most pregnant women do not have a deficiency of this mineral. Magnesium sulfate would not be effective against constipation in pregnant women. Magnesium sulfate does not stimulate musculoskeletal tone to augment labor contractions; rather, it has the opposite effect.

If a pregnant woman in labor calls the health care facility, the nurse should strongly advise the woman to come to the facility to be evaluated. a. True b. False

b. False If the initial contact is made by phone the nurse needs to ask woman about her s/s and what she is experiencing. Nurse then instructs woman to remain at home or come to the facility based on the woman's responses Fetal movement?

During a spontaneous vaginal birth several things need to occur to the fetus in sequence. As the fetus encounters resistance, what is its usual reaction? a. Extension b. Flexion c. internal rotation d. engagement

b. Flexion

At 24 weeks, a patient's BP rose from a pre-pregnant baseline of 120/70 to 150/92. No other problematic signs and symptoms or proteinuria were noted. a. Chronic hypertension b. Gestational hypertension c. Mild preeclampsia d. Severe preeclampsia

b. Gestational hypertension

A patient has been hypertensive since 24 weeks gestation. Urinalysis indicates a protein content of 3+. Further testing reveals a platelet count of 95,000 and elevated AST and ALT levels. She has begun to experience nausea and epigastric pain a. Eclampsia b. HELLP syndrome c. Severe Preeclampsia d. Chronic hypertension

b. HELLP syndrome

5. A pregnant woman, approximately 12 weeks' gestation, comes to the emergency department after calling her health care provider's office and reporting moderate vaginal bleeding. Assessment reveals cervical dilation and moderately strong abdominal cramps. She reports that she has passed some tissue with the bleeding. The nurse interprets these findings to suggest which of the following? a. Threatened abortion b. Inevitable abortion c. Incomplete abortion d. Missed abortion

b. Inevitable abortion An inevitable abortion is characterized by vaginal bleeding that is greater than slight, rupture of membranes, cervical dilation, strong abdominal cramping, and possible passage of products of conception. The threatened abortion involves slight vaginal bleeding, no cervical dilation and no change in cervical consistency, mild abdominal cramping, a closed cervical os, and no passage of fetal tissue. An incomplete abortion involves intense abdominal cramping, heavy vaginal bleeding, and cervical dilation. A missed abortion involves the absence of contractions and irregular spotting with possible progression to inevitable abortion.

Which fetal lie is most conducive to a spontaneous vaginal birth? a. Transverse b. Longitudinal c. Perpendicular d. Oblique

b. Longitudinal A longitudinal lie places the fetus in a vertical position, which would be the most conducive for a spontaneous vaginal birth. A transverse lie does not allow for a vaginal birth because the fetus is lying perpendicular to the maternal spine. A perpendicular lie describes the transverse lie, which would not be conducive for a spontaneous birth. An oblique lie would not allow for a spontaneous vaginal birth because the fetus would not fit through the maternal pelvis in this side-lying position.

Assessment reveals that the fetus of a client in labor is in the vertex presentation. The nurse determines that which part is presenting? a. Shoulders b. Occiput c. Brow d. buttocks

b. Occiput

As the nurse is explaining the difference between true versus false labor to her childbirth class, she states that the major difference between them is: a. Discomfort level is greater with false labor. b. Progressive cervical changes occur in true labor. c. There is a feeling of nausea with false labor. d. There is more fetal movement with true labor.

b. Progressive cervical changes occur in true labor. Progressive cervical changes occur in true labor. This is not the case with false labor.

A woman in labor at the hospital has just received an epidural block. Which intervention is a priority before and during epidural placement? a. Increase oral fluids every hour to prevent dehydration. b. Provide adequate IV fluids to maintain her blood pressure. c. Monitor temperature every four hours, and give acetaminophen if 100.4 degrees or higher. d. Monitor the maternal apical pulse for bradycardia.

b. Provide adequate IV fluids to maintain her blood pressure.

The laboring client who is at 3 cm dilation and 25% effaced is asking for analgesia. The nurse explains the analgesia usually is not administered prior to the establishment of the active phase. What is the appropriate rationale for this practice? a. This would cause fetal depression in utero. b. This may prolong labor and increase complications. c. The effects would wear off before birth. d. This can lead to maternal hypertension.

b. This may prolong labor and increase complications.

During the fourth stage of labor, the nurse assesses the woman at frequent intervals after giving childbirth. What assessment data would cause the nurse the most concern? a. Moderate amount of dark red lochia drainage on peripad b. Uterine fundus palpated to the right of the umbilicus c. An oral temperature reading of 100.6°F d. Perineal area bruised and edematous beneath her ice pack

b. Uterine fundus palpated to the right of the umbilicus A full bladder causes displacement of the uterus above it, and increased bleeding results secondary to the uncontracted status of the uterus. Massaging the uterus will help to make it firm but will not help to bring it back into the midline, since the full bladder is occupying the space the uterus would normally assume. Notifying the primary HCP is not necessary unless the woman continues to have difficulty voiding and the uterus remains displaced. The normal location of the uterus in the fourth stage of labor is in the midline. Displacement suggests a full bladder, which is not considered a normal finding.

A client who requested "no drugs" in labor asks the nurse what other options are available for pain relief. The nurse reviews several options for nonpharmacologic pain relief, and the client thinks effleurage may help her manage the pain. This indicates that the nurse will: a. lead the client through a series of visualizations to aid in relaxation. b. instruct the client or her partner to perform light fingertip repetitive abdominal massage. c. instruct the client to perform controlled chest breathing with a slow inhale and a quick exhale. d. press down firmly with her index finger and forefinger on key trigger points on the client's ankle or wrist.

b. instruct the client or her partner to perform light fingertip repetitive abdominal massage.

A client in labor has been admitted to the labor and birth suite. The nurse assessing her notes that the fetus is in a cephalic presentation. Which description should the nurse identify by the term presentation? a. relationship of the presenting part to the maternal pelvis b. part of the fetal body entering the maternal pelvis first c. relation of the fetal presenting part to the maternal ischial spine d. relation of the different fetal body parts to one another

b. part of the fetal body entering the maternal pelvis first

A client has presented in the early phase of labor. She is experiencing abdominal pain and shows signs of growing anxiety about the pain. What is the best pain management technique the nurse can suggest at this stage? a. immersing the client in warm water in a pool or hot tub b. practicing effleurage on the abdomen c. administering a sedative such as secobarbital or pentobarbital d. administering an opioid such as meperidine or fentanyl

b. practicing effleurage on the abdomen

The health care provider is evaluating a high-risk woman for a continuous internal monitoring. It would be most appropriate to meet which criterion? a. insertion by any staff b. rupture of membranes c. cervical dilation of 1 cm d. the presenting fetal part not visible

b. rupture of membranes

cephalohematoma

blood can collect beneath the scalp is a collection of blood between the periosteum and the bone that occurs several hours after birth. It does not cross suture lines and is generally reabsorbed over the next 6 to 8 weeks

4. After teaching a woman about hyperemesis gravidarum and how it differs from the typical nausea and vomiting of pregnancy, which statement by the woman indicates that the teaching was successful? a. "I can expect the nausea to last through my second trimester." b. "I should drink fluids with my meals instead of in between them." c. "I need to avoid strong odors, perfumes, or flavors." d. "I should lie down after I eat for about 2 hours."

c. "I need to avoid strong odors, perfumes, or flavors." The woman should avoid noxious stimuli such as strong flavors, odors, or perfumes because they might trigger nausea and vomiting.

A woman at 39 weeks gestation has been in labor for 8 hours and is asking how far she is dilated. She attended childbirth classes and is aware of the stages and phases of labor. She had a vaginal exam 30 minutes prior to her asking again. How should the nurse respond to her question? a. "I can arrange for a cervix check, if you want." b. "The health care provider will have to check you, we can call her." c. "Your labor signs have not changed; we are looking for changes in your labor pattern before we check you again." d. "Checking your cervix will not speed up labor; let's wait."

c. "Your labor signs have not changed; we are looking for changes in your labor pattern before we check you again."

What is the normally accepted fetal heart rate range? a. 90-140 bpm b. 100-150 bpm c. 110-160 bpm d. 120-170 bpm

c. 110-160 bpm

*The nurse should initially implement which intervention when a nulliparous woman telephones the hospital to report that she is in labor?* a. Tell the woman to stay home until her membranes rupture. b. Emphasize that food and fluid should stop or be light. c. Ask the woman to describe why she believes that she is in labor. d. Arrange for the woman to come to the hospital for labor evaluation.

c. Ask the woman to describe why she believes that she is in labor.

Which assessment would indicate that a woman is in true labor? a. Membranes are ruptured and fluid is clear. b. Presenting part is engaged and not floating. c. Cervix is 4 cm dilated, 90% effaced. d. Contractions last 30 seconds, every 5 to 10 minutes.

c. Cervix is 4 cm dilated, 90% effaced. True labor is characterized by contractions occurring at regular intervals that increase in frequency, duration, and intensity. These contractions bring about progressive cervical dilation and effacement. Thus, a cervix dilated to 4 cm and 90% effaced indicates true labor. Rupture of membranes may occur before the onset of labor, at the onset of labor, or at any time during labor and thus is not indicative of true labor. Engagement occurs when the presenting part reaches 0 station; it typically occurs 2 weeks before term in primigravidas and several weeks before the onset of labor or at the beginning of labor for multiparas. Contractions of true labor typically last 30 to 60 seconds and occur approximately every 4 to 6 minutes.

A client received IV meperidine for pain. An hour later a full-term neonate is born with respiratory depression. The nurse anticipates the neonate will require administration of which medication? a. indomethacin b. ampicillin c. naloxone d. epinephrine

c. naloxone

A woman is admitted to the ED with a medical diagnosis of ruptured ectopic pregnancy. The primary nursing diagnosis at this time would be: a. Acute pain R/T irritation of the peritoneum with blood b. Risk for infection R/T tissue trauma c. Deficient fluid volume R/T blood loss 2* rupture of the uterine tube d. Anticipatory grieving R/T unexpected pregnancy outcome

c. Deficient fluid volume R/T blood loss 2* rupture of the uterine tube a, b, and d are appropriate nursing diagnoses, but deficient fluid is the most immediate concern, placing the woman's well-being at greatest risk!

The nurse notes the presence of transient fetal accelerations on the fetal monitoring strip. Which intervention would be most appropriate? a. Reposition the client on the left side. b. Begin 100% oxygen via face mask. c. Document this as indicating a normal pattern. d. Call the health care provider immediately.

c. Document this as indicating a normal pattern. Fetal accelerations denote an intact CNS and appropriate oxygenation levels demonstrated by an increase in heart rate associated with fetal movement. Accelerations are a reassuring pattern, so no intervention is needed. Turning the woman on her left side would be an appropriate intervention for a late deceleration pattern. Administering 100% oxygen via face mask would be appropriate for a late or variable deceleration pattern. Since fetal accelerations are a reassuring pattern, no orders are needed from the health care provider, nor does the health care provider need to be notified of this reassuring pattern.

There are advantages and disadvantages to any kind of method used to control pain during labor and birth. What is an advantage of opioid administration? a. It provides total pain control. b. It is generally given p.o. c. It has the ability to be administered by the nurse. d. It can be given frequently without risk to the fetus.

c. It has the ability to be administered by the nurse.

Physiologic preparation for labor would be demonstrated by: a. Decrease in Braxton Hicks contractions felt by mother b. Weight gain and increase in appetite by mother c. Lightening, whereby the fetus drops into true pelvis d. Fetal heart rate accelerations and increased movements

c. Lightening, whereby the fetus drops into true pelvis since as labor nears, the fetus gets into position by descending into the maternal true pelvis in preparation for birth. The woman will experience heaviness in her lower pelvis and urinary frequency when this occurs. Response "A" is incorrect since there is an increase in uterine contractions as the uterus becomes more irritable and readies for true labor. Response "B" is incorrect since most women experience a weight loss and a decrease in appetite close to the start of their labor. Response "D" is incorrect due to the fact that the fetus is in a cramped environment at term and has limited room to move around. The fetal heart rate would remain within the normal range of 110 to 160 bpm unless there is a problem.

A pregnant client is admitted to a maternity clinic for birth. Which assessment finding indicates that the client's fetus is in the transverse lie position? a. Long axis of fetus is at 60° to that of client. b. Long axis of fetus is parallel to that of client. c. Long axis of fetus is perpendicular to that of client. d. Long axis of fetus is at 45° to that of client.

c. Long axis of fetus is perpendicular to that of client.

At 30 weeks gestation, a patient's blood pressure was 152/94; urinalysis indicated a protein level of 32mg/dl on dipstick; biceps and patellar reflexes are 2+. a. Eclampsia b. Severe Preeclampsia c. HELLP syndrome d. Mild Preeclampsia

c. Mild Preeclampsia

General anesthesia is not used frequently in obstetrics because of the risks involved. There are physiologic changes that occur during pregnancy that make the risks of general anesthesia higher than it is in the general population. What is one of those risks? a. The client is more sensitive to preanesthetic medications. b. The client is less sensitive to inhalation anesthetics. c. Neonatal depression is possible. d. Fetal hypersensitivity to anesthetic is possible.

c. Neonatal depression is possible.

A client is in active labor. As one of the nursing diagnoses is "Risk for trauma to the woman or fetus related to intrapartum complications or a full bladder," what would be appropriate for the nurse to do in order to achieve the goal of "no complications due to a full bladder"? a. Place a Foley catheter into the bladder. b. Get the woman up to void every two hours. c. Palpate the area above the symphysis pubis every two hours. d. Do a sterile "in and out" catheterization every two hours.

c. Palpate the area above the symphysis pubis every two hours.

A patient at 32 weeks comes to the ED with bright red vaginal bleeding. She reports no pain. The admission nurse suspects: a. Abruptio placentae b. Disseminated intravascular coagulation c. Placenta previa d. Preterm labor

c. Placenta previa the clinical manifestations of placenta previa are described dark red bleeding with pain is characteristic of abruptio placentae massive bleeding from many sites is associated with DIC bleeding is not a sign of preterm labor.

6. When administering magnesium sulfate to a client with preeclampsia, the nurse explains to her that this drug is given to: a. Reduce blood pressure b. Increase the progress of labor c. Prevent seizures d. Lower blood glucose levels

c. Prevent seizures prevent progression of preeclampsia into seizures. Magnesium sulfate is administered to prevent initial or recurrent seizures, while antihypertensives are used to reduce the maternal blood pressure to a safe range. Response "A" is incorrect since magnesium sulfate is not an antihypertensive drug. Typically, hydralazine, labetalol, or nifedipine is administered to lower maternal blood pressure. Response "B" is incorrect because magnesium sulfate does not impact the progress of labor, and doesn't speed it up. Response "D" is incorrect because it doesn't impact blood glucose levels.

A 32-year-old woman presents to the labor and birth suite in active labor. She is multigravida, relaxed, and talking with her husband. When examined by the nurse, the fetus is found to be in a cephalic presentation. His occiput is facing toward the front and slightly to the right of the mother's pelvis, and he is exhibiting a flexed attitude. How does the nurse document the position of the fetus? a. LOA b. LOP c. ROA d. ROP

c. ROA

When managing a client's pain during labor, nurses should: a. Make sure the agents given do not prolong labor b. Know that all pain relief measures are similar c. Support the client's decisions and requests d. Not recommend nonpharmacologic methods

c. Support the client's decisions and requests The entire focus of the labor and birth experience is for the family to make decisions, not the caretakers. The nurse's role is to respect and support those decisions. Decisions about pain management are not based on length of the various stages of labor, but rather on what provides effective pain relief for the laboring woman. Pain relief measures differ. Each individual responds differently and uniquely to various pain relief measures. Not recommending nonpharmacologic measures demonstrates bias on the nurse's part; it is not the nurse's decision to make, but rather the client's.

The shortest but most intense phase of labor is the: a. Latent phase b. Active phase c. Transition phase d. Placental expulsion phase

c. Transition phase The transition phase of the 1st stage of labor occurs when the contractions are 1 to 2 minutes apart and the final dilation is taking place. The transition phase is the most difficult and, fortunately, the shortest phase for the woman, lasting approximately 1 hour in the first birth and perhaps 15 to 30 minutes in successive births. Many women are not able to cope well with the intensity of this short period, become restless, and request pain medications. During the latent phase, contractions are mild. The woman is in early labor and able to cope with the infrequent contractions. This phase can last hours. The active phase involves moderate contractions that allow for a brief rest period in between, helping the woman to be able to cope with the next contraction. This phase can last hours. The placental expulsion phase occurs during the third stage of labor. After separation of the placenta from the uterine wall, continued uterine contractions cause the placenta to be expelled. Although this phase can last 5 to 30 minutes, the contraction intensity is less than that of the transition phase.

Which of the following observations would suggest that placental separation is occurring? a. Uterus stops contracting altogether. b. Umbilical cord pulsations stop. c. Uterine shape changes to globular. d. Maternal blood pressure drops.

c. Uterine shape changes to globular. After the placenta separates from the uterine wall, the shape of the uterus changes from discoid to globular. The uterus continues to contract throughout the placental separation process and the umbilical cord continues to pulsate for several minutes after placental separation occurs. Maternal BP is not affected by placental separation because the maternal blood volume has increased dramatically during pregnancy to compensate for blood loss during birth.

A nurse is conducting an in-service program for staff nurses working in the labor and birth unit. The nurse is discussing ways to promote a positive birth outcome for the woman in labor. The nurse determines that additional teaching is necessary when the group identifies which measure? a. promoting the woman's feelings of control b. providing clear information about procedures c. allowing the woman time to be alone d. encouraging the woman to use relaxation techniques

c. allowing the woman time to be alone

A woman is in the fourth stage of labor. During the first hour of this stage, the nurse would assess the woman's fundus at which frequency? a. every 5 minutes b. every 10 minutes c. every 15 minutes d. every 20 minutes

c. every 15 minutes

The RN in labor and birth documents the fetus as ROA. To what does this documentation refer for a fetus? a. fetal station b. fetal attitude c. fetal position d. fetal size

c. fetal position

By the end of the second stage of labor, the nurse would expect which of the following events? The a. cervix is fully dilated and effaced b. placenta is detached and expelled c. fetus is born and on mother's chest d. woman to request pain medication

c. fetus is born and on mother's chest because the second stage of labor is defined as beginning with complete dilation of the cervix (10 cm) and ending with the expulsion of the fetus. Response "A" is incorrect because the cervix is fully dilated at the start of stage 2, not at the end of it. Response "B" is incorrect due to the fact that the third stage of labor is defined as the period following the birth of the newborn through the expulsion of the placenta. Response "D" is incorrect since typically most women desiring pain medication would be requesting it in the active phase of labor in stage 1.

A nurse performs an initial assessment of a laboring woman and reports the following findings to the primary care provider: fetal heart rate is 152 bpm, cervix is 100% effaced and 5 cm dilated, membranes are intact, and presenting part is well applied to the cervix and at -1 station. The nurse recognizes that the client is in which stage of labor? a. Second b. first, latent c. first, active d. third

c. first, active

The nurse is determining how often contractions occur measuring from the beginning of the one contraction to the beginning of the next contraction. The nurse documents this finding as: a. duration. b. intensity. c. frequency. d. peak.

c. frequency.

To give birth to her infant, a woman is asked to push with contractions. Which pushing technique is the most effective and safest? a. lying supine with legs in lithotomy stirrups b. squatting while holding her breath c. head elevated, grasping knees, breathing out d. lying on side, arms grasped on abdomen

c. head elevated, grasping knees, breathing out

The most serious complication of hydatidiform mole is the development of ___________________ afterward.

choriocarcinoma

A pregnant client is admitted to a maternity clinic after experiencing contractions. The assigned nurse observes that the client experiences pauses between contractions. The nurse knows that which event marks the importance of the pauses between contractions during labor? a. effacement and dilation of the cervix b. shortening of the upper uterine segment c. reduction in length of the cervical canal d. restoration of blood flow to uterus and placenta

d. restoration of blood flow to uterus and placenta

A client is having a routine prenatal visit and asks the nurse what the birth education teacher meant when she used the term zero station. What is the best response by the nurse? a. "This is just a way of determining your progress in labor." b. "This indicates that you start labor within the next 24 hours." c. "This means +1 and the baby is entering the true pelvis." d. "The presenting part is at the true pelvis and is engaged."

d. "The presenting part is at the true pelvis and is engaged."

Assessment of a woman in labor reveals that the scapula of the fetus is the presenting part. The nurse interprets this finding as indicating which fetal presentation? a. Cephalic b. Vertex c. Breech d. shoulder

d. shoulder

A fetus is assessed at 2 cm above the ischial spines. How would the nurse document the fetal station? a. +4 b. +2 c. 0 d. -2

d. -2

As a woman enters the second stage of labor, her membranes spontaneously rupture. When this occurs, what would the nurse do next? a. Test a sample of amniotic fluid for protein. b. Ask her to bear down with the next contraction. c. Elevate her hips to prevent cord prolapse. d. Assess fetal heart rate for fetal safety.

d. Assess fetal heart rate for fetal safety.

The client is in the second stage of labor and is at 2+ station and in anticipation of birth within the hour. Her epidural did not work, and she is begging for meperidine. Which is the most appropriate action by the nurse? a. At the nurses' station call the anesthetist to retry the epidural. b. Call the primary care provider, and obtain a reduced dose of meperidine. c. Give the meperidine because she needs pain relief now. d. Encourage her through the contractions, and explain why she is not receiving the meperidine.

d. Encourage her through the contractions, and explain why she is not receiving the meperidine.

A pregnant client is admitted to a maternity clinic for birth. The client wishes to adopt the kneeling position during labor. The nurse knows that which to be an advantage of adopting a kneeling position during labor? a. It helps the woman in labor to save energy. b. It facilitates vaginal examinations. c. it facilitates external belt adjustment. d. It helps to rotate fetus in a posterior position

d. It helps to rotate fetus in a posterior position

What is the most important thing a nurse can do during labor and birth to prevent maternal and fetal infection? a. Clean the woman's perineum with a Betadine scrub. b. Strictly follow universal precautions. c. Remove soiled drapes and linen; place an absorbent pad under the buttocks and two sterile perineal pads against the perineum. d. Thoroughly wash the hands before and after client contact.

d. Thoroughly wash the hands before and after client contact.

When a client in labor is fully dilated, which instruction would be most effective to assist her in encouraging effective pushing? a. Hold your breath and push through entire contraction. b. Use chest-breathing with the contraction. c. Pant and blow during each contraction. d. Wait until you feel the urge to push.

d. Wait until you feel the urge to push. since nondirected pushing, based on current research, leads to better outcomes for both mother and infant. Holding breath and pushing throughout the entire contraction reduce blood flow and oxygenation to the fetus. Chest breathing is not effective since it doesn't increase abdominal pressure to assist the uterus to contract. Panting and blowing are used to abstain from pushing, which is not what is needed to expel the fetus.

A nurse is caring for a pregnant client in labor in a health care facility. The nurse knows that which sign marks the termination of the first stage of labor in the client? a. diffuse abdominal cramping b. rupturing of fetal membranes c. start of regular contractions d. dilation of cervix diameter to 10 cm

d. dilation of cervix diameter to 10 cm

A woman is lightly stroking her abdomen in rhythm with her breathing during contractions. The nurse identifies this technique as: a. acupressure. b. patterned breathing. c. therapeutic touch. d. effleurage.

d. effleurage.

The nurse is assisting a client through labor, monitoring her closely, now that she has received an epidural. The nurse would report which finding to the anesthesiologist? a. dry, cracked lips b. urinary retention c. rapid progress of labor d. inability to push

d. inability to push

What is a nursing intervention that helps prevent the most frequent side effect from epidural anesthesia in a pregnant client? a. administrating IV ephedrine b. administrating IV naloxone c. maintaining the client in a supine position d. starting an IV and hanging IV fluids

d. starting an IV and hanging IV fluids

During the second stage of labor, a woman is generally: a. very aware of activities immediately around her. b. anxious to have people around her. c. no longer in need of a support person. d. turning inward to concentrate on body sensations.

d. turning inward to concentrate on body sensations.

*medical intervention for ectopic pregnancy*

early diagnosis, most women could be treated with *methotrexate*, the overall success rate of medical treatment in properly selected women is nearly 90% To be eligible for medical therapy, the client must be hemodynamically stable, with no signs of active bleeding in the peritoneal cavity, low beta-hCG levels (<5,000 mIU/mL) and the mass (which must measure less than 4 cm as determined by ultrasound) must be unruptured. Contraindications to medical treatment include an unstable client, severe persistent abdominal pain, renal or liver disease, immunodeficiency, active pulmonary disease, peptic ulcer, suspected intrauterine pregnancy, and poor client compliance. The potential advantages include avoidance of surgery, the preservation of tubal patency and function, and a lower cost. The medical approach today for an unruptured tubal pregnancy most often consists of a single-dose IM injection of methotrexate (Rheumatrex; Trexall) with outpatient follow-up. Medical management with methotrexate, though not approved by the Food and Drug Administration for this purpose, has been endorsed by the American College of Obstetricians and Gynecologists. Prostaglandins, misoprostol, and actinomycin have also been used in the medical (nonsurgical) management of ectopic pregnancy, with a reported success rate of approximately 90%. Methotrexate is a folic acid antagonist that inhibits cell division in the developing embryo. It typically has been used as a chemotherapeutic agent in the treatment of leukemia, lymphomas, and carcinomas. It has been shown to produce results similar to that for surgical therapy in terms of high success rate, low complication rate, and good reproductive potential (Dalia et al., 2015). Adverse effects associated with methotrexate include nausea, vomiting, stomatitis, diarrhea, gastric upset, increased abdominal pain, and dizziness. Methotrexate for an ectopic pregnancy is ordered based on the client's body surface area. The administration of methotrexate should be limited to people who have had education and training in the handling and administration of hazardous drugs. In some facilities internal policies require that only nurses who have completed a chemotherapy certification and/or hazardous drug competency program can administer this product. Prior to receiving the single-dose intramuscular injection to treat unruptured pregnancies, the woman needs to be counseled on the risks, benefits, adverse effects, and the possibility of failure of medical therapy, which would result in tubal rupture, necessitating surgery. The woman is then instructed to return weekly for follow-up laboratory studies for the next several weeks until beta-hCG titers decrease. Beta-hCG level changes between days 0 and 4 after methotrexate therapy have clinical significance and predictive value. A decreasing beta-hCG level is highly predictive of treatment success

A pregnancy in which the blastocyst implants outside the uterus is an ___________________ pregnancy.

ectopic

*standards of nursing care* how it affects nursing practice, and the legalities of practicing using these standards

establish minimum criteria for competent, proficient delivery of nursing care.

caput succedaneum

fluid can also collect in the scalp can be described as edema of the scalp at the presenting part. This swelling crosses suture lines and disappears within 3 to 4 days. Reassure parents that it will go away IT crosses over the suture lines so it will go around the whole head like a cap Difference in a suffilent hematoma- doesn't cross over suture line 1 side, pooling of blood, dark, edema

*HELLP Syndrome!!!*

hemolysis elevated liver enzymes low platelet count - It is a variant of the preeclampsia/eclampsia syndrome that occurs in 10% to 20% of clients whose conditions are labeled as severe - increased risk for complications: cerebral hemorrhage, retinal detachment, hematoma/liver rupture, acute renal failure, DIC, placental abruption and maternal death - It is a life-threatening obstetric complication considered by many to be a severe form of preeclampsia involving hemolysis, thrombocytopenia, and liver dysfunction - Both HELLP and preeclampsia occur during the later stages of pregnancy, and sometimes after childbirth - is a clinically progressive condition - Early diagnosis is critical to prevent liver distention, rupture, and hemorrhage and the onset of DIC - If the condition presents prenatally, morbidity and mortality can affect both mother and baby. - occurs in up to *20% of pregnant women diagnosed with severe preeclampsia* - It is unique, as it is a laboratory-value specific diagnosis. - usually have fewer signs of abnormalities consistent with the metabolic syndrome and a lower prevalence of thrombophilia as compared with preeclampsia women without HELLP - has been reported as early as 17 weeks' gestation, most of the time it is diagnosed between 22 and 36 weeks' gestation - can present prior to the presence of an elevated blood pressure - leads to an increased maternal risk for developing liver hematoma or rupture, stroke, cardiac arrest, seizure, pulmonary edema, DIC, subendocardial hemorrhage, adult respiratory distress syndrome, renal damage, sepsis, hypoxic encephalopathy, and maternal or fetal death - recognition of HELLP syndrome and an aggressive multidisciplinary approach and prompt transfer of these women to obstetric centers with expertise in this field are required for the improvement of maternal-fetal prognosis.

Cindy, a 20-year-old primipara, calls the birthing center where you work as a nurse and reports that she thinks she is in labor because she feels labor pains. Her due date is this week. The midwives have been giving her prenatal care throughout this pregnancy. a. What additional information do you need to respond appropriately?

i. Ask about the frequency and duration of her contractions. ii. Ask about how long she has experienced "labor pains." iii. Ask about any other signs she may have experienced such as bloody show, lightening, backache, ruptured membranes, and so forth. iv. Ask if walking tends to increase or decrease the intensity of contractions. v. Ask her when she last felt fetal movement. vi. Ask her how far away (distance) she is from the birthing center. vii. Ask her if she has a support person in the home with her.

What other premonitory signs of labor might the nurse ask about?

i. Has she experienced the feeling of the fetus dropping (lightening) lower down? ii. Has her energy level changed (increased) in the last day or so? iii. Has she noticed any reddish discharge (bloody show) from her vagina? iv. Has she had any episodes of diarrhea within the last 48 hours? v. Has her "bag of waters" broken or does she feel any leakage?

You are assigned to lead a community education class for women in their third trimester of pregnancy to prepare them for their upcoming birth. Prepare an outline of topics that should be addressed. a. Topics to address in the community education program would include:

i. Information about the stages of labor, including what to expect ii. Explanation of risks and benefits about any interventional procedures that might be performed during the labor process iii. Information about the available pain relief measures iv. Methods of involvement and participation during the labor and birthing process by partner/doula/family member v. Information about variables that may alter or influence the course of labor, including preoperative teaching for cesarean birth

What instructions need to be given to guide her decision making?

i. Instruct her on how to time the frequency and duration of contractions. ii. Wait until contractions are 5 minutes apart or her membranes rupture to come to the birthing center. iii. *Tell her to come to the birthing center when she cannot talk during a contraction.* iv. Reinforce all instructions with her support partner.

A 20-year-old primigravida at term, comes to the birthing center in active labor (dilation 5 cm and 80% effaced, -1 station) with ruptured membranes. She states she wants an "all-natural" birth without medication. Her partner is with her and appears anxious but supportive. On the admission assessment, this client's prenatal history is unremarkable; vital signs are within normal limits; FHR via Doppler ranges between 140 and 144 bpm and is regular. a. Based on your assessment data and the woman's request not to have medication, what nonpharmacologic interventions could you offer her? b. What positions might be suggested to facilitate fetal descent?

i. Progressive relaxation techniques of locating, then releasing tension from one muscle group at a time until the entire body is relaxed ii. Visual imagery such as taking a journey in the woman's mind to a relaxing place that is far away from the discomfort of labor iii. Music to bring about a calming effect as well as a distraction or attention focusing to divert attention away from the laboring process; focusing on sound or rhythm helps release tension and promote relaxation iv. Massage/acupressure to enhance relaxation, improve circulation, and reduce pain in labor; counterpressure on the lower back to help relieve back pain v. Breathing techniques for effective attention-focusing strategies to enhance coping mechanisms during labor i. Upright positions such as walking, swaying, slow-dancing with her partner, or leaning over a birthing ball will all enhance comfort and use the force of gravity to facilitate fetal descent. ii. Kneeling and leaning forward will help relieve back pain. iii. Pelvic rocking on hands and knees and lunging with one foot elevated on a chair may help with internal fetal rotation and speed a slow labor.

What suggestions/recommendations would you make to her?

i. Stay in the comfort of her home environment as long as possible. ii. Advise her to walk as much as possible to see what effect it has on the contractions. Also, tell her to drink fluids to hydrate herself. iii. Review nonpharmacologic comfort measures she can try at home. iv. Tell her to keep in contact with the birthing center staff regarding her experience.

Several hours later, the client complains of nausea and turns to her partner and angrily tells him to not touch her and to go away. a. What assessment needs to be done to determine what is happening? b. What explanation can you offer Carrie's partner regarding her change in behavior?

i. The nurse should perform a vaginal examination to validate that Carrie is in the transition phase (8 to 10 cm dilated). i. Explain to her partner that she is in the transition phase of the first stage of labor and that her behavior is typical, since she is having hard contractions frequently. Reassure him not to take Carrie's comments personally, but to stay and be supportive to her.

ANDROID PELVIS

is considered the male-shaped pelvis and is characterized by a funnel shape. It occurs in approximately 20% of women - The pelvic inlet is heart-shaped and the posterior segments are reduced in all pelvic planes. - Descent of the fetal head into the pelvis is slow, and failure of the fetus to rotate is common. - The prognosis for labor is poor, subsequently leading to cesarean birth.

GYNECOID PELVIS

is considered the true female pelvis, occurring in about 40% of all women; it is less common in men - vaginal birth is most favorable with this type of pelvis because the inlet is round and the outlet is roomy - This shape offers the optimal diameters in all three planes of the pelvis - this type of pelvis allows early and complete fetal internal rotation during labor.

PLATYPELLOID (FLAT) PELVIS

is the least common type of pelvic structure among men and women, with an approximate incidence of 3% - the pelvic cavity is shallow but widens at the pelvic outlet, making it difficult for the fetus to descend through the mid-pelvis - Labor prognosis is poor with arrest at the inlet occurring frequently - It is not favorable for a vaginal birth unless the fetal head can pass through the inlet. Women with this type of pelvis usually require cesarean birth. An important principle is that most pelvises are not purely defined but occur in nature as mixed types. Many women have a combination of these four basic pelvis types, with no two pelves being exactly the same. Regardless of the shape, the newborn can be born vaginally if size and positioning remain compatible. The narrowest part of the fetus attempts to align itself with the narrowest pelvic dimension (e.g., biparietal to interspinous diameters, which means the fetus generally tends to rotate to the most ample portion of the pelvis).

*placental abruption clinical manifestations*

o Placental abruption refers to premature separation of a normally implanted placenta from the maternal myometrium. Placental abruption occurs in about 1% of all pregnancies throughout the world and is associated with significant perinatal mortality and morbidity (March of Dimes, 2015e). Risk factors include preeclampsia, gestational hypertension, seizure activity, advanced maternal age >34, uterine rupture, trauma, smoking, cocaine use, coagulation defects, chorioamnionitis, premature rupture of membranes, hydramnios, uterine trauma, external cephalic version for breech presentation, previous history of abruption, domestic violence, and placental pathology. These conditions may force blood into the underlayer of the placenta and cause it to detach o Management of placental abruption depends on the gestational age, the extent of the hemorrhage, and maternal-fetal oxygenation perfusion/reserve status. Treatment is based on the circumstances. Typically once the diagnosis is established, the focus is on maintaining the cardiovascular status of the mother and developing a plan to deliver the fetus quickly. A cesarean birth may take place quickly if the fetus is still alive with only a partial abruption. A vaginal birth may take place if there is fetal demise secondary to a complete abruption.

*lightening*

occurs when the fetal presenting part begins to descend into the true pelvis. - fetus descends into pelvis "dropped" - When mom feels like she can breathe again but it causes her to have to pee more

*fetal presentation!*

refers to the body part of the fetus that enters the pelvic inlet first (the "presenting part"). This is the fetal part that lies over the inlet of the pelvis or the cervical os. Knowing which fetal part is coming first at birth is critical for planning and initiating appropriate interventions. - the 3 main fetal presentations are: 1. *cephalic /vertex (head first)* 2. breech (pelvis first) 3. shoulder (scapula first) - The majority of term newborns (95%) enter this world in a cephalic presentation; breech presentations account for 3% of term births, and shoulder presentations for approximately 2% - in a cephalic presentation, the presenting part is usually the occipital portion of the fetal head - Variations in a vertex presentation include the military, brow, and facial presentations

*fetal lie!*

refers to the relationship of the long axis (spine) of the fetus to the long axis (spine) of the mother. There are three possible lies: longitudinal (which is the most common), transverse (Fig. 13.6), and oblique. A longitudinal lie occurs when the long axis of the fetus is parallel to that of the mother (fetal spine to maternal spine side-by-side). A transverse lie occurs when the long axis of the fetus is perpendicular to the long axis of the mother (fetal spine lies across the maternal abdomen and crosses her spine). In an oblique lie, the fetal long axis is at an angle to the bony inlet, and no palpable fetal part is presenting. This lie is usually transitory and occurs during fetal conversion between other lies. A fetus in a transverse or oblique lie position cannot be delivered vaginally

fetal station

relation of the presenting part to ischial spines of the maternal pelvis Measured in cm - Positive at the bottom "we're positive the babies on the way out" At 0 station - the babies head is engaged when the widest part of the babies head is entered into the mid-pelvis When fetal head is pushed on vaginal floor and can see head crowning up on contractions = crowning Failure to descend

*clinical manifestations of HELLP*

similar to severe preeclampsia - Be alert for complaints of: nausea (with or without vomiting) (50%) malaise (90%) epigastric or RUQ pain (65%) demonstrable edema - diagnosis is made based on laboratory test results, including: Low hematocrit that is not explained by any blood loss Elevated LDH (liver impairment) Elevated AST (liver impairment) Elevated ALT (liver impairment) Elevated BUN Elevated bilirubin level Elevated uric acid and creatinine levels (renal involvement) Low platelet count (less than 100,000 cells/mm3)

dilation

the opening or enlargement of the external cervical os

*fetal attitude!*

the relation of the fetal body parts to each other - refers to the posturing (flexion or extension) of the joints and the relationship of fetal parts to one another - most common when labor begins is with all joints flexed—the fetal back is rounded, the chin is on the chest, the thighs are flexed on the abdomen, and the legs are flexed at the knees; this normal fetal position is most favorable for vaginal birth, presenting the smallest fetal skull diameters to the pelvis - when the fetus presents to the pelvis with abnormal attitudes (no flexion or extension), their nonflexed position can increase the diameter of the presenting part as it passes through the pelvis, increasing the difficulty of birth - an attitude of extension tends to present larger fetal skull diameters, which may make birth difficult

fetal position

the relation of the presenting part to the four quadrants of the mother's *pelvis* - The 1st letter defines whether the presenting part is tilted toward the left (L) or the right (R) side of the maternal pelvis. - The 2nd letter represents the particular presenting part of the fetus: O for occiput, S for sacrum (buttocks), M for mentum (chin), A for acromion process, and D for dorsal (refers to the fetal back) when denoting the fetal position in shoulder presentations - The 3rd letter defines the location of the presenting part in relation to the anterior (A) portion of the maternal pelvis or the posterior (P) portion of the maternal pelvis. If the presenting part is directed to the side of the maternal pelvis, the fetal presentation is designated as transverse (T). ROA: right occiput anterior ROP: right occiput posterior ROT: right occiput transverse LOT: left occiput transverse *LOA*: left occiput anterior LOP: left occiput posterior OP "sunnyside up" babys face is looking up at the ceiling Take Note! *LOA is the most common (and most favorable) fetal position for birthing today, followed by right occiput anterior (ROA). The positioning of the fetus allows the fetal head to contour to the diameters of the maternal pelvis. LOA and ROA are optimal positions for a vaginal birth.*

A client has just given birth to a healthy baby boy, but the placenta has not yet delivered. What stage of labor does this scenario represent? a. first b. second c. third d. fourth

third

*ABO incompatibility!*

type O mothers & fetuses with type A or B blood (less severe than Rh incompatibility)

*bloody show*

vaginal mucus becomes more profuse in response - brownish or blood tinged cervical mucus may be passed. - may just mean shes passed her mucus plug, not in labor

*Braxton Hicks Contractions*

which the woman may have been experiencing throughout the pregnancy, may become stronger and more frequent - are typically felt as a tightening or pulling sensation of the top of the uterus - They occur primarily in the abdomen and groin and gradually spread downward before relaxing. - usually last about 30 seconds but can persist for as long as 2 minutes - Take Note! *An infant born between 34 0/7 and 36 6/7 weeks of gestation is identified as "late preterm" and experiences many of the same health issues as other preterm birth infants*

*Mild Preeclampsia*

- BP: >140/90 mm Hg after 20 weeks' gestation - Proteinuria: 300 mg/24 hr or greater than 1+ protein on a random dipstick urine sample - Seizures/coma: No - Hyperreflexia: No - Other signs and symptoms: Mild facial or hand edema Weight gain

*BOX 7-3 Nursing Malpractice Prevention*

1. *Failure to ensure patient safety* i. Monitor patients in a timely manner consistent with agency policy and the changing needs of the patient. Assess and document potential for injury. Incorporate safety needs into the plan of care. ii. Clearly define criteria for use of restrictive devices. Ensure that the use of restrictive devices is consistent with agency policy. Use the least restrictive devices that will be effective in preventing injury. iii. Update your knowledge involving patient safety and new interventions to prevent and reduce injury. iv. Evaluate whether patients at high risk for injury are routinely identified before injury results. 2. *Improper treatment or performance of treatment* i. Question treatments that you believe are improper. Know your agency's policy for questioning a problematic order. ii. Use proper techniques when performing procedures, and follow agency procedures. iii. Seek assistance when unsure of a new procedure. Never perform an intervention until you know what you are doing, why you are doing it, and your ongoing assessment and teaching responsibilities. iv. Update your clinical skills through continuing education classes, conferences, and workshops. 3. *Failure to monitor and report* i. Follow physician orders regarding monitoring of patients unless changes in the patient's condition necessitate a change in the frequency of monitoring; report the need for change to the physician. ii. Report any requested information or significant changes in a patient's condition. If unsure of the significance of an observed change, consult with an experienced colleague. iii. Perform appropriate and timely nursing assessments. iv. Ensure that the nurse-patient ratio is adequate. 4. *Med errors and reactions* i. Verify any questionable medical orders. ii. Verify patient's name and date of birth before administering medication and use appropriate agency-specific identifiers such as bar coding. iii. Listen to any patient questions or objections regarding a medication, and investigate the patient's concerns before administering the medication. iv. Refer to a drug reference for any questions about appropriate dosages, side effects, and reactions. v. Know your agency's policies on verbal and written medication orders and on medication administration. vi. Update your knowledge of medications and new medication administration protocols. 5. *Failure to follow agency procedure* i. Know your agency's procedures. Ensure that your orientation to new responsibilities familiarizes you with pertinent policies and procedures. ii. If you must deviate from a procedure, discuss the incident with your supervisor and decide on appropriate action. iii. Advise the appropriate person of procedures that need to be revised. 6. *Documentation* i. Document significant information about your patients objectively and factually. ii. Know and follow the agency's documentation policies. iii. Document specific times that you performed actions, made observations, or performed patient assessments. iv. Document legibly when writing, spell correctly, and use only agency-approved abbreviations. v. Be sensitive to privacy considerations when documenting on a computer. vi. Routinely evaluate the quality of documentation and update your knowledge of new documentation methodologies. 7. *Equipment use* i. Learn how to operate equipment in a safe and appropriate manner. Never operate equipment with which you are unfamiliar. ii. Use predetermined procedures when teaching patients how to use equipment, and ensure that all nurses involved in client education are teaching the same procedures. iii. Provide home care patients with the telephone number of a 24-hour backup hospital or home care service available in case of emergency. iv. Have patients demonstrate their competence with equipment before allowing them to use it. v. Attend orientations and in-services on the use of new or modified equipment. 8. *Adverse incidents* i. If an adverse incident occurs, complete the appropriate documentation and report the incident to the designated person following agency policy. ii. Do not assume, voice, or record any blame for the incident. iii. Know the institutional chain of command for reporting instances when patient care is at issue. iv. Support agency loss prevention programs that identify potential liabilities, guard against patient injuries, and maximize the defense of the agency and its employee nurses. 9. *Clients w/ HIV* u. Be conscious of actions that could result in a lawsuit: i. Discrimination in treatment ii. Nosocomial (in-hospital) transmission of virus iii. Breach of confidentiality iv. Participation in testing a patient for HIV without first obtaining informed consent v. Know and follow agency policies and procedures for the care of patients with infectious diseases. w. Update your knowledge of HIV infection; be familiar with national standards (such as those established by the Centers for Disease Control and Prevention) and pertinent state/province laws.

*mgmt of gestational HTN*

A recent study found that progesterone supplementation during the first trimester significantly reduced the incidence of gestational hypertension and fetal distress in primigravida women. This supplementation might be a future therapy with addition studies to validate it

*Severe Preeclampsia*

>160/110 mm Hg >500 mg/24 hr; greater than 3+ on random dipstick urine sample - Seizures/coma: No - Hyperreflexia: No - Other s/s: Headache Oliguria Blurred vision, scotomata (blind spots) Pulmonary edema Thrombocytopenia (platelet count <100,000 platelets/mm3) Cerebral disturbances Epigastric or RUQ pain HELLP

*Eclampsia*

>160/110 mm Hg Marked proteinuria - Seizures/coma: Yes - Hyperreflexia: Yes - other s/s: Severe headache Generalized edema RUQ or epigastric pain Visual disturbances Cerebral hemorrhage Renal failure HELLP

certified nurse midwife (CNM)

A RN who holds at least a master's degree in nursing and advanced levels of education in the management of maternity. - Certification is achieved through an organized program of study and national testing by the American College of Nurse-Midwives. - means the performance for compensation of advanced nursing practices by a certified nurse midwife that are relevant to the management of women's health care, focusing on pregnancy, childbirth, the postpartum period, care of the newborn, family planning, and gynecological needs of women, within a healthcare system that provides for consultation, collaborative management, or referral as indicated by the health status of the client - Hold current certification by a national certifying body recognized by the Board in the advanced practice registered nurse role and population foci appropriate to educational preparation; and Have an agreement with a consulting physician if providing intrapartum care - has postgraduate training in the care of normal pregnancy and delivery and is certified by the American College of Nurse Midwives (ACNM) - Midwives are PCPs for women with a special emphasis on pregnancy, childbirth, and reproductive health - are committed to providing ethical, individualized, evidence-based care for all women throughout their life cycle

*Disseminated Intravascular Coagulation (DIC)*

- is a complex condition that leads to activation of coagulation; it usually occurs in critically ill children. Common triggers of DIC include septic shock, presence of endotoxins and viruses, tissue necrosis or injury, and cancer treatment (Ambruso et al., 2014). In DIC, thrombin is generated, fibrin is deposited in the circulation, and platelets are consumed. Deficiencies of coagulation and anticoagulation pathways occur. Hemorrhage and organ tissue damage result and can be irreversible if not recognized and treated immediately. Therapeutic management of children with DIC requires careful consideration of the etiology. Initial treatment focuses on treating the underlying cause. For example, if DIC occurs secondary to an infection, appropriate antibiotics would be used to treat the infection. Heparin is also used at lower doses to counteract the deficiency in the coagulation/anticoagulation pathway. Heparin reduces consumption of the platelets, resulting in improved platelet counts. Since heparin is an anticoagulant, there is an increased risk of bleeding.

*understand the use of the professional organizations in women's health and the guidance that they provide for nursing care*

1. ANA 2. AWHONN 3. NANN

BOX 1-4 ANA STANDARDS OF NURSING PRACTICE

1. Assessment: The registered nurse collects comprehensive data pertinent to the patient's health or the situation. 2. Diagnosis: The registered nurse analyzes the assessment data to determine the diagnoses or issues. 3. Outcomes Identification: the registered nurse identifies expected outcomes for a plan individualized to the patient or the situation. 4. Planning: The registered nurse develops a plan that prescribes strategies and alternatives to attain expected outcomes. 5. Implementation: The registered nurse implements the identified plan, coordinates care delivery, employs strategies to promote health and a safe environment (the advanced practice registered nurse [APRN] also provides consultation and uses prescriptive authority and treatment). 12. Standard 5a. Coordination of Care 13. The registered nurse coordinates care delivery. 14. Standard 5b. Health Teaching and Promotion 15. The registered nurse employs strategies to promote health and a safe environment. 16. Standard 5c. Consultation 17. The graduate-level prepared specialty nurse or APRN provides consultation to influence the identified plan, enhance the abilities of others, and effect change. 18. Standard 5d. Prescriptive Authority and Treatment 19. The APRN uses prescriptive authority, procedures, referrals, treatments, and therapies in accordance with state and federal laws and regulations. 20. Standard 6. Evaluation 21. The registered nurse evaluates progress toward attainment of outcomes. 22. Standards of Professional Performance 23. Standard 7. Ethics 24. The registered nurse practices ethically. 25. Standard 8. Education 26. The registered nurse attains knowledge and competence that reflects current nursing practice. 27. Standard 9. Evidence-Based Practice and Research 28. The registered nurse contributes to quality nursing practice. 29. Standard 10. Quality of Practice 30. The registered nurse contributes to quality nursing practice. 31. Standard 11. Communication 32. The registered nurse communicates effectively in all areas of practice. 33. Standard 12. Leadership 34. The registered nurse demonstrates leadership in the professional practice setting and the profession. 35. Standard 13. Collaboration 36. The registered nurse collaborates with the health care consumer, family, and others in the conduct of nursing practice. 37. Standard 14. Professional Practice Evaluation 38. The registered nurse evaluates her or his own nursing practice in relation to professional practice standards and guidelines, relevant statutes, rules, and regulations. 39. Standard 15. Resource Utilization 40. The registered nurse utilizes appropriate resources to plan and provide nursing services that are safe, effective, and financially responsible. 41. Standard 16. Environmental Health 42. The registered nurse practices in an environmentally safe and healthy manner. 43. Standard 17. Collegiality 44. The nurse interacts with and contributes to the professional development of peers and colleagues.

*determine resources that a new nurse can their increase knowledge level about policies and procedures in the healthcare setting*

1. state's nurse practice act 2. hospital's policies and procedures 3. ANA 4. NLN 5. International Council of Nurses (ICN) - Professional nursing organizations in the United States include the American Nurses Association (ANA), the National League for Nursing (NLN), and the American Association of Colleges in Nursing (AACN). The National Student Nurses' Association (NSNA) prepares students to participate in professional nursing organizations. - The ANA defines continuing education as those professional development experiences designed to enrich the nurse's contribution to health. Colleges, hospitals, voluntary agencies, and private groups offer formal continuing education through courses, seminars, and workshops. In many states, continuing education is required for an RN to maintain licensure.

*CHART 3-4 American Nurses Association Code of Ethics for Nurses*

1. the nurse, in all professional relationships, practices with *compassion and respect for the inherent dignity, worth, and uniqueness of every individual, unrestricted by considerations of social or economic status, personal attributes, or the nature of health problems* 2. The nurse's primary commitment is to the *patient*, whether an individual, family, group, or community. 3. The nurse promotes, advocates for, and strives to *protect* the health, safety, and rights of the patient. 4. The nurse is responsible and accountable for individual nursing practice and determines the appropriate *delegation* of tasks consistent with the nurse's obligation to provide optimum patient care. 5. The nurse *owes the same duties to self as to others*, including the responsibility to preserve integrity and safety, to maintain competence, and to continue personal and professional growth. 6. The nurse participates in establishing, maintaining, and improving *health care environments* and conditions of employment conducive to the provision of quality health care and consistent with the values of the profession through individual and collective action. 7. The nurse participates in the *advancement* of the profession through contributions to practice, education, administration, and knowledge development. 8. The nurse *collaborates* with other health professionals and the public in promoting community, national, and international efforts to meet health needs. 9. The profession of nursing, as represented by associations and their members, is responsible for *articulating nursing values, for maintaining the integrity of the profession and its practice, and for shaping social policy*

preterm premature rupture of membranes (PPROM)

Break in fetal membranes before term (less than 37 weeks' gestation). women <37 weeks' gestation Prolonged

infant mortality rate

A measure of the rate of deaths of liveborn infants before their first birthday; the numerator is the number of infants less than 1 year of age born alive in a defined region during a calendar year who die before they are 1 year old; the denominator is the total number of live births. Childbirth in colonial America was a difficult and dangerous experience. During the 17th and 18th centuries, women giving birth often died as a result of exhaustion, dehydration, infection, hemorrhage, or seizures (Foster, 2015). Approximately 50% of all children died before age 5 (Norman, 2015), compared with the 0.06% infant mortality rate of toda - The infant mortality rate is the number of deaths occurring in the first 12 months of life - *Take Note! Non-Hispanic, African American infants have consistently had higher infant mortality rates than other ethnic groups*

*mature minor*

Person <18 years of age, who nonetheless possesses an understanding of the nature and consequences of proposed treatment. In some states, may give consent to certain medical treatment. - HCP must determine that the adolescent (usually >14 years of age) is sufficiently mature and intelligent to make the decision for treatment. - HCP also considers the complexity of the treatment, its risks and benefits, and whether the treatment is necessary or elective before obtaining consent from a mature minor

morbidity

A diseased state. The ratio of sick to well people in a community. SEE ALSO morbidity rate The frequency of the appearance of complications following a surgical procedure or other treatment. - is the measure of the prevalence of a specific illness in a population at a particular time. It is presented in rates per 1,000 population. Morbidity is often difficult to define and record because the definitions used vary widely. For example, morbidity may be defined as visits to the physician or diagnosis for hospital admission. Also, data may be difficult to obtain. Morbidity statistics are revised less frequently because of the difficulty in defining or obtaining the information.

doula

A woman who assists at labor and birth and in postpartum care of mother and baby. Doulas are trained and certified according to various requirements of local jurisdictions. They are helpful in educating the new family and in helping build their confidence as new parents. - In many ways, childbirth practices in the United States have come full circle, as we see the return of nurse midwives and doulas. Today, childbirth choices are often based on what works best for the mother, child, and family. - is a nonmedical birth companion who provides continuous emotional, physical, and educational support to the woman and family during childbirth and the postpartum period. Doulas do not perform clinical or medical tasks; they are there to comfort and support the mother and to enhance communication between the mother and medical professionals (DONA International, n. d.). Many nurses working in labor and birth areas today are credentialed in their specialty so that they can provide optimal care to the woman and her newborn.

*emancipated minor*

A young person who is legally entitled to be treated as an adult through a court order, marriage, military service, or being a parent. - state laws vary in relation to this definition and the types of treatment that may be obtained by an emancipated minor (without parental consent). o The nurse must be familiar with the particular state's law. o This may be considered in any of the following situations, depending on the state's laws: Membership in a branch of the armed services Marriage Court-determined emancipation Financial independence and living apart from parents Pregnancy Mother less than 18 years of age - s considered to have the legal capacity of an adult and may make his or her own health care decision - many states do not require consent or notification of parents or legal guardians when providing specific care to minors - depending on the state law, health care may be provided to minors for certain conditions, in a confidential manner, without including the parents, these types of care may include pregnancy counseling, prenatal care, contraception, testing for and treatment of sexually transmitted infections and communicable diseases (including HIV), substance abuse, and mental illness counseling and treatment - These exceptions allow children to seek help in a confidential manner in situations where they might otherwise avoid care if they were required to inform their parents or legal guardian - *laws vary by state, so the nurse must be knowledgeable about the laws in the state where he or she is licensed to practice*

*advance directives*

Advance directives determine the child's and family's wishes should life-sustaining care become necessary - Parents are generally the surrogate decision makers for children. If the child's interests are not served by prolonged survival, then the physician or advanced practitioner should educate the parents about the extent of the child's illness and potential for ongoing quality of life. After discussion with other family members, friends, and spiritual advisors, the parents may make the decision to forego life-sustaining medical treatment, either withdrawing treatment or deciding to withhold further treatment or opt not to resuscitate in the event of cardiopulmonary arrest. Life-sustaining care may include antibiotics, chemotherapy, dialysis, ventilation, cardiopulmonary resuscitation, and artificial nutrition and hydration. Some families may choose to withdraw these treatments if they are already in place or to not begin them should the need arise. Do not attempt resuscitation (DNAR) orders are in place for some children, particularly the terminally ill. - Health care providers must continue to work with parents of extremely sick or premature infants to ensure they are accurately informed about the condition of their child and the risks and benefits to treatment. Health care providers must also be aware of federal, state, and hospital policy regarding care of very ill, premature, and/or disabled newborns. The nurse must be knowledgeable about the laws related to health care of children in the state where he or she practices as well as the policies of the health care institution. The nurse must be sensitive to the various ethical situations that he or she may become involved in and should apply knowledge of laws as well as concepts of ethics to provide appropriate care.

*stem cells*

Any precursor cell. A cell with daughter cells that may differentiate into other cell types - constitute one of the most promising tools for regenerative medicine. They can grow into any cell type in the body and have been touted as a potential cure for everything from type 1 diabetes to stroke. The goal of stem cell research is a relief of human suffering, which ethically is good. Benefits of stem cell research include therapies for Parkinson disease, regeneration of diseased body tissues, repair of spinal cord injuries, and the growing of needed organs for transplant. It would seem morally compelling to explore all the sources that might provide us with them, but the ethical concerns surrounding stem cell research vary depending on the origin of the stem cells. Adult stem cells are found in adults, who can replace old cells by reproducing new ones (e.g., blood and liver cells). Bone marrow transplants are examples of the use of adult stem cells in medical therapy. Embryonic stem cells are derived from the inner cell mass of an early embryo. They aren't without controversy; the process of obtaining these cells results in the destruction of the embryo. However, they hold huge promise too Some people feel that the destruction of the human embryo constitutes the killing of a human being and reject this practice on religious grounds. Views about when life begins and whether the early embryo is considered a person with moral status are the heart of the ethical deliberations related to embryonic stem cells. Everyone has to decide based on their own personal values and ethical background.

*Emergency Medical Treatment & Active Labor Act (EMTALA)*

Appropriate/legal transfer criteria: 1. Treat and Stabilize Patient 2. Medically necessary to receive proper care 3. Physician documentation of necessity 4. Agreement to accept and treat patient 5. All records 6. Qualified personal accompany patient - require that every client who presents at an emergency department is given a medical examination regardless of informed consent or reimbursement ability - every ED with a Medicare provider agreement must perform a medical screening examination on all patients arriving with an emergency medical complaint if their acute signs and symptoms could result in serious injury or death if left untreated. EDs are also required to provide treatment aimed at stabilizing each patient's condition. If the patient must be transferred to another facility, the patient's consent for transfer should be obtained, if possible. In addition, acceptance by the receiving facility and physician must be obtained, and an appropriate method of transfer for the patient should be secured. Documentation of assessment and treatment must be sent with the patient upon transfer

*do not attempt resuscitate (DNAR) orders*

Directive to health care workers from a patient who has expressed in writing a wish not to be resuscitated in the event of cardiac or respiratory arrest. Some families may choose to withdraw these treatments if they are already in place or to not begin them should the need arise. These orders are in place for some children, particularly the terminally ill. o Some institutions have started using the term AND (Allow Natural Death). o No matter what term is used, these orders should include specific instructions regarding the child's and family's wishes (e.g., some families may desire oxygen but not chest compressions or code medications). o When the child is hospitalized, the order must be documented in the physician orders and updated according to the facility's policy. o These orders may also be in place in the home, but only a few states allow emergency medical services to honor a child's DNAR order in the home. o Children with DNAR orders may also still be attending school. o In that case, the health care professionals involved should meet with the school officials (the board of education and its legal counsel) to discuss how the DNAR request can be upheld in the school setting o The health care provider should help educate the school about the child's condition, potential complications, and health care goals o They should work with the school and family on developing an individualized health care plan that will include what to do instead of CPR such as comfort measures

punishment

Discipline is often confused with punishment, but punishment involves a negative or unpleasant experience or consequence for doing or not doing something. Although punishment is sometimes a necessary element of discipline, to be an effective tool it must be coupled with rewards for good behavior - Punishment may be verbal or corporal. Verbal punishment commonly takes the form of reprimands or scolding (the use of disapproving statements). The statements are intended to change or eliminate the inappropriate behavior. Verbal reprimands can be effective in the short term if they are used sparingly and are focused on the child's specific behavior. If verbal reprimands are used frequently and indiscriminately, they lose their effectiveness, can provoke anxiety in the child, and encourage the child to ignore the parent. Frequent use also may reinforce the behavior by providing the child with attention - *Take Note! Other forms of discipline, if used incorrectly, can also cause problems for the child and interfere with the caregiver-child relationship. For example, disapproval using tone of voice, facial expression, or gestures can be effective in stopping inappropriate behavior, but if the disapproval comes in the form of verbal statements that attack the child rather than the behavior, negative consequences may occur*

polyhydramnios

Excess amount of amniotic fluid. - Amniotic fluid >2,000 mL Therapeutic Management: close monitoring; removal of fluid, indomethacin (decreases fluid by decreasing fetal urinary output) Nursing Assessment: risk factors, fundal height, abdominal discomfort, difficulty palpating fetal parts or obtaining FHR Nursing Management: ongoing assessment and monitoring; assisting with therapeutic amniocentesis

foster care

Families today face complex challenges as they try to nurture, develop, and socialize their members. Family structure changes such as divorce, blending families, adoption, or foster care can have wide-ranging and lifelong effects. These special situations require astute assessment and proactive intervention to minimize the risk to the family.

*gestational hypertension*

Hypertension during pregnancy in a previously normotensive woman or aggravation of hypertension during pregnancy in a hypertensive woman. SYN pregnancy-induced hypertension

*understand what situations are occurring that fall under the topic of ethical issues/dilemmas in women's health*

Ethical issues are increasing as technology increases and includes: 1. General issues of health and wellness of women 2. Genetic and embryo research 3. Conception and reproduction 4. Pregnancy and maternal/fetal issues 5. Neonate health and wellness 6. Contraception and Abortion - until fairly recently, the fetus was viewed legally as a non-person. Mother and fetus were viewed as one complex client - the pregnant woman - of which the fetus was an essential part. - These interventions infringe on the autonomy of the mother. Attempts have been made to criminalize the behavior of women who fail to follow a physician's advice or who engage in risky behaviors that are considered harmful to the fetus. - In 2005, Arkansas passed a bill to include in the definition of neglect the causing of a newborn child to be born with an illegal substance in his or her system or born with a health problem as a result of the pregnant mother's use before birth of an illegal substance. The mother is then reported to the state for possible child abuse. Only HCP are acceptable to make these reports. Other states have similar laws, so you need to know the law in the state in which you practice [Garrets Law!!!] - Refuses interventions on behalf of the fetus Forced intervention may include: 1. Forced cesarean birth 2. Coercion of mothers who practice high-risk behaviors (substance abuse) 3. Mandating experimental in-utero therapy or surgery to correct birth defects 4. Intrauterine fetal surgery, Fetoscopy 5. Therapeutic insemination 6. Genetic engineering, Stem cell research 7. Surgery for infertility 8. "Test tube" babies 9. Treatment of very-low-birth-weight (VLBW) babies 10. Surrogate motherhood 11. Sex selection 12. Selective reduction of multiple pregnancies 13. Termination of pregnancy following prenatal diagnosis 14. Cloning - Several areas are of particular importance to the health care of women and children. These include abortion, substance abuse, fetal therapy, maternal-fetal conflict, stem cell research, umbilical cord blood banking, informed consent, client rights, and confidentiality.

*understand the importance of ethical integrity as a nurse and the use of the ANA Code of Ethics for Nurses*

In the ANA Social Policy Statement and Scope and Standards of Practice, nursing is defined as "the protection, promotion, and optimization of health and abilities, prevention of illness and injury, alleviation of suffering through the diagnosis and treatment of human response, and advocacy in the care of individuals, families, communities, and populations." - This definition supports the claim that nurses must be actively involved in the decision-making process regarding ethical concerns surrounding health care and human responses - Nurses are morally obligated to present ethical conflicts within a logical, systematic framework - Health care settings in which nurses are valued members of the team promote interdisciplinary communication and may enhance patient care - To practice effectively in these settings, nurses must be aware of ethical issues and assist patients in voicing their moral concerns. - Nurses have a responsibility to carry out their role as described in the Social Policy Statement (ANA) to comply with the nurse practice act of the state in which they practice, and to comply with the Code of Ethics for Nurses as spelled out by the ANA and the ICN - To have a foundation for examining the delivery of nursing care, it is necessary to understand the needs of health care consumers and the health care delivery system, including the forces that affect nursing and health care delivery. integrity: adherence to moral and ethical principles; soundness of moral character; honesty.

*National Association of Neonatal Nurses (NANN)*

Informed Consent: 1. Protects a client's right to autonomy and self-determination 2. Influences ability to give consent: a. Anxiety b. Fear c. Pain d. Medications 3. Emergency situations - As used in this subdivision, "emergency situation" means a situation in which, in competent medical judgment, the proposed surgical or medical treatment or procedures are immediately or imminently necessary and any delay occasioned by an attempt to obtain a consent would reasonably be expected to jeopardize the life, health, or safety of the person affected or would reasonably be expected to result in disfigurement or impaired faculties -today, more than ever before, high-risk neonates have a better chance of thriving because of the advances in medicine and the dedicated care they receive. Neonatal nurses are on the front line, providing around-the-clock care for these most vulnerable patients - NANN stands behind these nurses, supporting and tending to their educational and professionals needs. - NANN is a community of registered nursing professionals at all stages of their careers who care for newborn infants born with a variety of health challenges, including prematurity, birth defects, infection, cardiac malformations, and surgical problems.

*intrauterine therapy*

Progress in prenatal diagnosis can lead to the diagnosis of severe fetal abnormalities for which previously one would anticipate a fatal outcome or the development of severe disability despite optimal postnatal care - Intrauterine therapy can now be offered in these selected cases and also in the treatment of fetal obstructive uropathy, intrauterine transfusions for fetal anemia, spina bifida repair, and stem cell transplantation - Intrauterine therapy is a procedure that involves opening the uterus during pregnancy, performing a surgery, and replacing the fetus in the uterus. Although the risks to the fetus and the mother are both great, fetal therapy may be used to correct anatomic lesions (Yuan, 2015). Some argue that medical technology should not interfere with nature, and thus this intervention should not take place. Others would argue that the surgical intervention improves the child's quality of life. For many people, these are the subjects of debates and intellectual discussions, but for nurses, these procedures may be part of their daily routine. Nurses play an important supportive role in caring and advocating for clients and their families. As the use of technology grows, situations will surface more frequently that test a nurse's belief system. Encouraging open discussions to address emotional issues and differences of opinion among staff members is healthy and increases tolerance for differing points of view.

premature rupture of membranes (PROM)

Rupture of the amnionic sac before onset of labor. women >37 weeks' gestation Treatment: dependent on gestational age; no unsterile digital cervical exams until woman is in active labor; expectant management if fetal lungs immature Nursing Assessment: risk factors, signs and symptoms of labor, electronic FHR monitoring, amniotic fluid characteristics, Nitrazine test, fern test, ultrasound Nursing Management Infection prevention Identification of uterine contractions Education and support Discharge home (PPROM) if no labor within 48 hours

*substance abuse*

Substance abuse for any person is a problem, but when it involves a pregnant woman, substance abuse can cause fetal injury and thus has legal and ethical implications - Substance abuse during pregnancy may cause fetal anomalies, preterm birth, placenta abruption, and central nervous system developmental alterations (Viteri et al., 2015). Many state laws require reporting evidence of prenatal drug exposure, which may lead to charges of negligence and child endangerment against the pregnant woman. It has been found that incarceration, or threat of it, has no effect in reducing cases of alcohol or drug abuse (Malinowska-Semprunch & Rychkova, 2015). This punitive approach to fetal injury raises ethical and legal questions about the degree of governmental control that is appropriate in the interests of child safety. All pregnant women and women of childbearing age should be screened periodically for alcohol, tobacco, and prescription and illicit drug use. Nurses should employ a flexible approach to the care of women who have substance use problems, and they should encourage the use of all available community resources. Nurses should counsel women about the risks of preconception, antepartum, and postpartum substance abuse in a calm, nonjudgmental manner

*American Nurses Association (ANA)*

The ANA is the professional organization for RNs in the United States. Founded in the late 1800s, its membership is comprised of the state nurses' associations to which individual nurses belong. Its primary mission is to be involved in public education, clinical nursing standards, and lobbying state and federal lawmakers to advance the profession of nursing. The ANA addresses ethics, public policy, and the economic and general welfare of nurses.

National League for Nurses (NLN)

The NLN is an organization open to all people interested in nursing, including nurses, nonnurses, and agencies. Established in 1952, its objective is to foster the development and improvement of all nursing services and nursing education. The NLN conducts one of the largest professional testing services in the United States, including pre-entrance testing for potential students and achievement testing to measure student progress. It also serves as the primary source of research data about nursing education, conducting annual surveys of schools and new RNs. The organization also provides voluntary accreditation for educational programs in nursing.

*health hx and physical exam for preeclampsia*

Take a thorough history during the first antepartal visit to identify whether the woman is at risk for preeclampsia. Risk factors include: Primigravida status Chromosomal abnormalities Structural congenital anomalies Multiple gestation History of preeclampsia in a previous pregnancy Excessive placental tissue, as is seen in women with GTD Chronic stress Use of ovulation drugs Family history of preeclampsia (mother or sister) Lower socioeconomic status History of diabetes, hypertension, or renal disease Poor nutrition Lower socioeconomic status African-American ethnicity Age extremes (younger than 20 or older than 35) Obesity - In addition, complete a nutritional assessment that includes the woman's usual intake of protein, calcium, daily calories, and fluids. Women at risk for preeclampsia require more frequent prenatal visits throughout their pregnancy, and they require teaching about problems so that they can report them promptly. Blood pressure must be measured carefully and consistently. Obtain all measurements with the woman in the same position (blood pressure is highest in the sitting position and lowest in the side-lying position) and by using the same technique (automated vs. manual). This standardization in position and technique will yield the most accurate readings. Obtain the client's weight (noting gain since last visit), and assess for amount and location of edema. Asking questions such as "Do your rings still fit on your fingers?" or "Is your face puffy when you get up in the morning?" will help to determine whether fluid retention is present or if the woman's status has changed since her last visit. - *Take Note! Although edema is not a cardinal sign of preeclampsia, weight should be monitored frequently to identify sudden gains in a short time span. Current research relies less on the classic triad of symptoms (hypertension, proteinuria, and edema or weight gain) and more on decreased organ perfusion, endothelial dysfunction (capillary leaking and proteinuria), and elevated blood pressure as key indicators* - If edema is present, assess the distribution, degree, and pitting. Document your findings and identify whether the edema is dependent or pitting. Dependent edema is present on the lower half of the body if the client is ambulatory, where hydrostatic pressure is greatest. It is usually observed in the feet and ankles or in the sacral area if the client is on bed rest. Pitting edema is edema that leaves a small depression or pit after finger pressure is applied to a swollen area. Record the depth of pitting demonstrated when pressure is applied. Although subjective, the following is used to record relative degrees: 1+ pitting edema = 2-mm depression into skin; disappears rapidly 2+ pitting edema = 4-mm skin depression; disappears in 10 to 15 seconds 3+ pitting edema = 6-mm depression into skin; lasts more than 1 minute 4+ pitting edema = 8-mm depression into skin; lasts 2 to 3 minutes At every antepartal visit, assess the fetal heart rate with a Doppler device. Also check a clean-catch urine specimen for protein using a dipstick. - *Take Note! The absolute blood pressure (value that validates elevation) of 140/90 mm Hg should be obtained on two occasions 4 to 6 hours apart to be diagnostic of preeclampsia. Proteinuria is defined as 300 mg or more of urinary protein per 24 hours or more than 1+ protein by chemical reagent strip or dipstick of at least two random urine samples collected at least 4 to 6 hours apart with no evidence of urinary tract infection*

*placenta previa*

The condition in which the placenta is implanted in the lower segment of the uterus, extending to the margin of the internal os of the cervix or partially or completely obstructing the os. SYN placental presentation - is a bleeding condition that occurs during the last two trimesters of pregnancy. In placenta previa (literally, "afterbirth first"), the placenta implants over the cervical os. It may cause serious morbidity and mortality to the fetus and mother. The risk of placenta previa in a first pregnancy is 1 in 400, but it rises to 1 in 160 after one cesarean section; 1 in 60 after two; 1 in 30 after three; and 1 in 10 after four cesarean sections and is associated with potentially serious consequences from hemorrhage, abruption (separation) of the placenta, or emergency cesarean birth. With the rising incidence of cesarean section operations combined with increasing maternal age and more infertility treatments, the number of cases of placenta previa is increasing dramatically. The cesarean section rate must be reduced to decrease maternal morbidity and mortality. Comprehensive risk assessment, combined with advances in ultrasound, can provide earlier detection of this impaired placental implantation - Total placenta previa: the internal cervical os is completely covered by the placenta Partial placenta previa: the internal os is partially covered by the placenta Marginal placenta previa: the placenta is at the margin or edge of the internal os Low-lying placenta previa: the placenta is implanted in the lower uterine segment and is near the internal os but does not reach it

*ectopic pregnancy*

The development of an impregnated oocyte *outside* the cavity of the uterus - is any pregnancy in which the *fertilized ovum implants outside the uterine cavity* - refers to the implantation of a fertilized egg in a location outside of the uterine cavity, including the fallopian tubes (95%), cervix, ovary, and the abdominal cavity - This abnormally implanted embryo grows and draws its blood supply from the site of abnormal implantation - As the embryo enlarges, it creates the potential for organ rupture because only the uterine cavity is designed to expand and accommodate fetal development - can lead to massive hemorrhage, infertility, or death. - occur in 1 in every 50 pregnancies in the US or roughly 2% of all pregnancies are diagnosed as ectopic; this rate has increased dramatically during the past 30 years - It is a major health concern for women of reproductive age and is the primary cause of death during the 1st trimester of pregnancy in the US - The discovery of these prior to rupture has increased dramatically in the past few decades as a result of improved diagnostic techniques such as the development of sensitive and specific radioimmunoassays for hCG, high-resolution ultrasonography, and the widespread availability of laparoscopy - rupture and hemorrhage may occur due to the growth of the embryo, *that fallopian tube will not be able to sustain that pregnancy * - A ruptured ectopic pregnancy is a *medical emergency*; therefore, prediction of any tubal rupture before its occurrence is extremely important - It is a potentially *life-threatening* condition and involves pregnancy loss - women can and do still die from it, although early diagnosis has helped prevent that - *Leading cause of infertility* - After surgical treatment: Recurrent rate of 10% - 25% - After methotrexate treatment: Recurrent rate of 11%

maternal mortality ratio

The number of maternal deaths occurring as the direct result of the reproductive process per 100,000 live births. - is the annual number of deaths from any cause related to or aggravated by pregnancy or its management (excluding accidental or incidental causes) during pregnancy and childbirth or within 42 days of termination of pregnancy, irrespective of the duration and site of the pregnancy, per 100,000 live births, for a specified year. In the United States, approximately 700 to 800 women die each year during pregnancy or shortly after childbirth. The rate of maternal mortality in the United States has more than doubled in the past few decades. A significant proportion of these deaths are preventable - In the United States, the maternal mortality ratio is mixed depending on ethnic background. African American women suffer maternal mortality ratios far higher than any other ethnic group. The risk of maternal mortality has remained about three to four times higher among Black women when compared to White women during the past six decades. About 42.8 of 100,000 African American mothers die due to childbirth, as compared to much lower rates in Whites (12.5) and Hispanics (8.9) (CDC, 2015f). The federal government has pledged to improve maternal-child care outcomes and thus reduce mortality ratios for women and children by endorsing the Healthy People 2020, but the WHO (2015c) data show that the United States ranks 48th in the world for maternal mortality. In fact, maternal mortality ratios are higher than almost all European countries, as well as several countries in Asia and the Middle East. For a country that spends more than any other country on health care and more on childbirth-related care than any other area of hospitalization, US $88 billion a year, this is a shockingly poor return on investment

child abuse and neglect

The physical, emotional, or sexual maltreatment of a child. Failure of a health care provider or caregiver to observe due care and diligence in performing services or delivering medicine or other products so as to avoid harming a patient. Generally, indifference or inadequate attention to one's responsibilities in regard to self-care, care of others, or other aspects of one's personal or professional life. "any recent act or failure to act on the part of a parent or caretaker that results in death, serious physical or emotional harm, sexual abuse or exploitation, or an act or failure to act that presents an imminent risk of serious harm to a child" In 2012, approximately 686,000 cases of child maltreatment occurred in the US. Of the number of children identified as being maltreated, 78% were victims of neglect, 18% were victims of physical abuse, 9% were victims of sexual abuse, and 11% were victims of other maltreatment, such as emotional or psychological abuse. Approximately 1,640 children died from abuse and neglect in 2012; nearly 70% of those children were less than 3 years of age - *Take Note! Children of African American and American Indian or Alaska Native descent have higher rates of abuse and neglect* - *Take Note! Witnessing and being exposed to violence in childhood results in a higher tolerance, and greater use, of violence as an adult*

Oligohydramnios

The presence of an insufficient amount of amnionic fluid (<500 mL at term) Amniotic fluid <500 mL Therapeutic Management: serial monitoring; amnioinfusion and birth for fetal compromise Nursing Assessment: risk factors, fluid leaking from vagina Nursing Management: continuous fetal surveillance; assistance with amnioinfusion, comfort measures, position changes

ethnocentrism

The tendency to evaluate other groups according to the values and standards of one's own ethnic group, especially with the conviction that one's own ethnic group is superior to other groups - Stereotyping or labeling can result from ethnocentrism, a belief that one's own ethnic group is superior to other ethnic groups. This attitude can lead to a slanted view of the world, and it may hinder the nurse's ability to provide culturally competent care. - *Take Note! Nurses can help to shape a person's lifelong perceptions of health and health services. An understanding of how the woman's, child's, and family's culture affects their health practices gives the nurse an opportunity to incorporate appropriate and beneficial health practices into the family's cultural milieu, providing sources of strength rather than areas of conflict.* - *Take Note! Never make assumptions about a family's religious or spiritual affiliation. Although they may belong to a particular religion, family members may not adhere to all of its beliefs or participate in all aspects of the religion. Be alert for clues that would provide insight into their specific beliefs.*

Charting by Exception (CBE)

a shorthand documentation method that makes use of well-defined standards of practice; only significant findings or "exceptions" to these standards are documented in narrative notes. i. Benefits of this approach include less time needed for charting (freeing more time for direct patient care), a greater emphasis on significant data, easy retrieval of significant data, timely bedside charting, standardized assessment, greater interdisciplinary communication, better tracking of important patient responses, and lower costs. ii. A significant drawback to charting by exception is its limited usefulness when trying to prove that high-quality safe care was given if a negligence claim is made against nursing. As more facilities move to a totally electronic medical record, even facilities not previously using a CBE system are considering doing so.

*confidentiality*

With the establishment of the *Health Insurance Portability and Accountability Act (HIPAA) of 1996*, the confidentiality of health care information is now mandated by law. - The primary intent of the law is to maintain health insurance coverage for workers and their families when they change or lose jobs - Another aspect of the law requires the Department of Health and Human Services to establish national standards for EMRs - With the increased use of EMRs and electronic billing came the increased possibility that personal health information might be inappropriately distributed. Client confidentiality and privacy must be maintained in the same manner as it is with paper documentation. Nurses can ensure that privacy is maintained when using computerized documentation and EMRs by doing the following: Always maintain the security of personal log-in information; nurses should never share it with other health care providers or other persons. Always log off when leaving the computer. Do not leave client information visible on a monitor screen when the computer/monitor is unattended. Use safeguards, such as encryption, when using alternative means of communication such as email. Exceptions to confidentiality exist. For example, suspicion of physical or sexual abuse and injuries caused by a weapon or criminal act must be reported to the proper authorities. Abuse cases are reported to the appropriate welfare authorities, whereas criminal acts are reported to the police. In addition, if the minor is a threat to self, then information may need to be disclosed to protect the child. The health care provider must also follow public health laws that require reporting certain infectious diseases to the local health department (e.g., tuberculosis, hepatitis, HIV, and other sexually transmitted infections). Finally, there is a duty to warn third parties when a specific threat is made to an identifiable person. Health care providers must strike a balance between confidentiality and required disclosure. Even if disclosure is required it is recommended that the health care provider attempt to gain consent for the disclosure and when possible inform the minor of the limits to confidentiality and consent prior to the initiation of care

*understand the importance of documentation and the use of the chain of command*

a. Documentation is crucial evidence of the quality of nursing care b. Must be complete, concise, accurate c. Charting by Exception (CBE) d. Chart what is happening in the room e. The patient record is the ONLY permanent legal document that details the nurse's interactions with the patient and is the nurse's best defense if a patient or patient surrogate alleges nursing negligence. f. Legal documentation: Patient records are legal documents that might be used as evidence in court proceedings. 1 in 4 malpractice suits are decided on the basis of the patient's record. Documentation plays an important role in implicating or absolving health care practitioners charged with improper care. The record can also be used in accident or injury claims made by the patient. No nurse can afford to be ignorant of or careless about agency policies and professional standards for documentation. g. Patient records serve many purposes. The ANA states that the most important of these is "communicating within the health care team and providing information for other professionals, primarily for individuals and groups involved with accreditation, credentialing, legal, regulatory and legislative, reimbursement, research, and quality activities"

*dissent*

disagreeing with the treatment plan, it may be ethically binding - if the HCP is not going to honor the pediatric client's dissent, then the argument can be made that the HCP should NOT ask for the client's assent - in some cases, such as in those of significant morbidity or mortality, this may need to be overridden; These cases need to be looked at on an individual basis - if the decision is made to move forward with treatment despite the child's dissent, then this decision must be explained to the child in developmentally appropriate terms. - there has been an increased emphasis on including children in research studies. - children are not little adults but 70% of medicines given to children have only been tested on adults - in research studies investigators, IRBs are responsible for ensuring measures are taken to protect the children in the studies. - *nurse caring for these children also has the responsibility to ensure protection at all stages of the research process* - nurses can become members of the IRB; they should also become familiar with studies that have been approved in their work setting to help ensure their pediatric clients are protected.

*AHA Patient' Bill of Rights*

i. All clients have the right to refuse medical treatment, based on the American Hospital Association's Bill of Rights. ii. The American Hospital Association first established a Patient's Bill of Rights in 1972 as a way to promote the value and dignity of the client. This information is updated periodically. Most health care agencies and professional organizations have developed some type of document that addresses client rights. iii. Ensuring that client rights are upheld is a key aspect in the care of any client. For the pregnant woman, two clients must be considered—the pregnant woman and her fetus. The American Foundation for Maternal and Child Health developed the Pregnant Patient's Bill of Rights to address specific concerns and situations involving the health and well-being of the mother and her fetus. In addition, the U.S. Department of Health and Human Services has issued laws related to protecting the welfare of newborns (the Baby Doe law). A child, due to his or her age and developmental level, may lack mature decision-making abilities. Many pediatric institutions have adopted a bill of rights for children's health care specific to that institution. This might include the following rights: 1. To be called by name 2. To receive compassionate health care in a careful, prompt, and courteous manner 3. To know the names of all providers caring for the child 4. To have basic needs met and usual schedules or routines honored 5. To make choices whenever possible 6. To be kept without food or drink when necessary for the shortest time possible 7. To be unrestrained if able 8. To have parents or other important persons with the child 9. To have an interpreter for the child and family when needed 10. To object noisily if desired 11. To be educated honestly about the child's health care 12. To be respected as a person (not having people talk about the child within earshot unless the child knows what is happening) 13. For all health care providers, to respect the child's confidentiality about his or her illness at all times iv. All clients have the right to refuse medical treatment based on the American Hospital Association's Bill of Rights. For a child, the parents have the legal right to decide for or against medical treatment. Minor children (under the age of 18 years) must have their parents or legal guardians provide consent for health care in most cases. v. The American Nurses Association's Bill of Rights for Registered Nurses is followed: "Nurses have the right to freely and openly advocate for themselves and their patients, without fear of retribution." vi. 2 of the chief reasons nurses cite for the declining quality of nursing care at their facilities are inadequate staffing and decreased nurse satisfaction. Advocacy on behalf of nurses and the profession has resulted in a tangible tool, the Bill of Rights for Registered Nurses, to aid in improving workplaces and ensuring nurses' ability to provide safe, quality patient care. The Bill of Rights is intended to empower nurses by making clear what is absolutely nonnegotiable in the workplace. "The ANA and its constituent member organizations will continue their longstanding work, through political and legislative activism, collective bargaining, workplace advocacy, and public education, to ensure that the basic tenets of this Bill of Rights are a reality for nurses across the country" vii. The seven basic tenets of the Bill of Rights for Registered Nurses are: 1. Nurses have the right to practice in a manner that fulfills their obligations to society and to those who receive nursing care. 2. Nurses have the right to practice in environments that allow them to act in accordance with professional standards and legally authorized scopes of practice. 3. Nurses have the right to a work environment that supports and facilitates ethical practice, in accordance with the Code of Ethics for Nurses and its interpretive statements. 4. Nurses have the right to freely and openly advocate for themselves and their patients, without fear of retribution. 5. Nurses have the right to fair compensation for their work, consistent with their knowledge, experience, and professional responsibilities. 6. Nurses have the right to a work environment that is safe for themselves and their patients. 7. Nurses have the right to negotiate the conditions of their employment, either as individuals or collectively, in all practice settings.

*gestational HTN assessment findings*

is characterized by hypertension (>140/90) without proteinuria after 20 weeks' gestation resolving by 12 weeks' postpartum - is defined as systolic blood pressure >140 mm Hg and/or diastolic >90 mm Hg on at least two occasions at least 4 to 6 hours apart after the 20th week of gestation, in women known to be normotensive prior to this time and prior to pregnancy

high-risk pregnancy

is one in which a condition exists that jeopardizes the health of the mother, her fetus, or both. The condition may result from the pregnancy, or it may be a condition that was present before the woman became pregnant.

Circulating Nurse

manages patient care in the Operating Room during a cesarean delivery.

*Assent*

means agreeing to something - in pediatric health care, the term refers to the child's participation in the decision-making process about health care - The age depends on the child's developmental level, maturity, and psychological state - children and adolescents be involved in the discussions about their health care and kept informed in an age-appropriate manner - As a child gets older this should be given more serious consideration - The pediatric client needs to be empowered by health care providers to the extent of their capabilities and as children mature and develop over time they should become the primary decision maker regarding their own health care - if a health care provider asks the child's opinion about the direction of treatment or participation in research, then the child's view and desires should be seriously considered - Take Note!: *Whenever possible, obtain assent for participation from the child.* - when obtaining this, 1st help the child to understand his or her health condition, depending on the child's developmental level - next, inform the child of the treatment planned and discuss what he or she should expect. - then determine what the child understands about the situation and make sure he or she is not being unduly influenced to make a decision one way or another. - finally, ascertain the child's willingness to participate in the treatment or research... this is a process and should continue throughout the course of treatment or research protocol.

The classification of hypertensive disorders in pregnancy currently consists of 5 categories:

1. Chronic hypertension: hypertension that exists prior to pregnancy or that develops before 20 weeks' gestation. 2. *Gestational hypertension*: blood pressure elevation (140/90 mm Hg) identified after 20 weeks' gestation without proteinuria. Blood pressure returns to normal by 12 weeks' postpartum. 3. *Preeclampsia*: most common hypertensive disorder of pregnancy, which develops with proteinuria after 20 weeks' gestation. It is a multisystem disease process, which is classified as mild or severe, depending on the severity of the organ dysfunction. 4. *Eclampsia*: Onset of seizure activity in a woman with preeclampsia. 5. Chronic hypertension with superimposed preeclampsia: occurs in approximately 20% of pregnant women with increased maternal and fetal morbidity rates

BOX 1.5 KEY ELEMENTS OF INFORMED CONSENT

1. The decision maker must be of legal age in that state, with full civil rights, and must be competent (have the ability to make the decision). 2. Present information that is simple, concise, and appropriate to the level of education and language of the individual responsible for making the decision. 3. The decision must be voluntary, and without coercion, force, or influence of duress. 4. Have a witness to the process of informed consent. 5. Have the witness sign the consent form.

*Association of Women's Health, Obstetrics, and Neonatal Nursing (AWHONN)*

have developed evidence-based clinical practice guidelines for the safest and effective delivery of family-centered nursing care Nurses should be diligent in seeking out these evidence-based guidelines to ensure excellence in their daily practice. When EBP is delivered in a context of caring and in a supportive organizational culture, the highest quality of care and best client outcomes can be achieved

enculturation

Culture (the view of the world and implementation of a set of traditions that are used by a specific social group in order to pass these traditions along to the next generation) plays a critical role in shaping a woman, child, or family including their health and health practices - Culture is a complex phenomenon involving the integration of many components such as beliefs, values, language, time, personal space, and view of the world, all of which shape a person's actions and behavior. Individuals learn these patterns of cultural behaviors from their family and community through a process called enculturation, which involves acquiring knowledge and internalizing values - Culture is learned first in the family, then in school, and then in the community and other social organizations. Culture influences every aspect of development and is reflected in childbearing and child-rearing beliefs and practices designed to promote healthy adaptation

preeclampsia

Development of hypertension with proteinuria or edema, or both, due to pregnancy or the influence of a recent pregnancy; it usually occurs after the 20th week of gestation, but may develop before this time in the presence of trophoblastic disease. - The "cure" for preeclampsia/eclampsia is always delivery of the placenta. The resolution following expulsion of the placenta supports theories related to the placental influence on the disease - According to recent studies, prevention of preeclampsia should be considered with daily low-dose aspirin from 12 weeks' gestation and onward to women identified at high risk for it. - While women with chronic hypertension or a personal history of preeclampsia should receive aspirin during pregnancy, further research should be ongoing to predict preeclampsia in low-risk women

Special Situations to Informed Consent

If the parent is not available, then the person in charge (relative, babysitter, or teacher) may give consent for emergency treatment if that person has a signed form from the parent or legal guardian allowing him or her to do so - During an emergency situation, a verbal consent via the telephone may be obtained - Two witnesses must be listening simultaneously and will sign the consent form, indicating that consent was received via telephone - Health care providers can provide emergency treatment to a child without consent if they have made reasonable attempts to contact the child's parent or legal guardian - In urgent or emergent situations, appropriate medical care never should be delayed or withheld due to an inability to obtain consent - *Certain federal laws, such as the Emergency Medical Treatment and Labor Act (EMTALA), require that every client who presents at an ER is given a medical examination regardless of informed consent or reimbursement ability*

cultural competence

Knowledge and understanding of another person's culture; adapting interventions and approaches to health care to the specific culture of the patient, family, and social group. - With today's changing demographic patterns, nurses must be able to incorporate cultural knowledge into their interventions so that they can care effectively for culturally diverse women, children, and families. They must be aware of the wide range of cultural traditions, values, and ethics that exist in the United States today. All nurses must establish cultural competence, the ability to apply knowledge about a client's culture so that health care interventions can be adapted to meet the needs of the client. Cultural competence refers to the process by which individuals and systems respond respectfully and effectively to people of all cultures, languages, ethnic backgrounds, disabilities, religions, sexual orientation, and other diversity factors in a manner that recognizes, and values the worth of individuals, families, and communities

*informed consent*

Most care given in a health care setting is covered by the initial consent for treatment signed when the person becomes a client at that office or clinic, or by the consent to treatment signed upon admission to the hospital or other inpatient facility - certain procedures, however, require a specific process of informed consent. Procedures that require informed consent include major and minor surgery; invasive procedures such as amniocentesis, internal fetal monitoring, lumbar puncture, or bone marrow aspiration; treatments placing the person at higher risk, such as chemotherapy or radiation therapy; procedures or treatments involving research; application of restraints; and photography involving the person. Generally, only people over the age of majority (18 years of age) can legally provide consent for health care. Because children are minors, the process of consent involves obtaining written permission from a parent or legal guardian. In cases requiring a signature for consent, usually the parent gives consent for care of children less than 18 years of age except in certain situations (see discussion that follows). - *Take Note! Never assume that the adult accompanying the child is the parent or legal guardian. Always clarify the relationship of the accompanying adult.* The informed consent process, which must be done before the procedure or specific care, addresses the legal and ethical requirement of informing the person about the procedure. It originates from the right of the child and family to direct their care and the ethical responsibility of health care providers to involve the child and family in health care decisions. Nurses should involve children and adolescents in the decision-making process to the extent possible, though the parent is still ultimately responsible for giving consent. The physician or advanced practitioner providing or performing the treatment and/or procedure is responsible for informing the child and family about the procedure and obtaining consent by providing a detailed description of the procedure or treatment, the potential risks and benefits, and alternative methods available. The nurse's responsibility related to informed consent includes the following: Ensuring that the consent form is completed with signatures from the client (or parents or legal guardians if the client is a child) Serving as a witness to the signature process Determining whether the client or parents or legal guardians understand what they are signing by asking them pertinent questions - laws vary from state to state. Nurses must become familiar with state laws as well as the policies and procedures of the health care agency. Treating children without obtaining proper informed consent violates their rights, and the physician and/or facility may be held liable for any damages

discipline

Much of parenting involves increasing desirable behavior and decreasing or eliminating undesirable behavior, a process generally known as discipline. There are various opinions in our society about the best or most effective method of discipline. Each child and family is unique. Discipline that works with one child or within one family may not work for another family. Discipline should focus on the development of the child while ensuring to preserve their self-esteem and dignity. It should be based on age-appropriate expectations with clear, consistent guidelines while offering meaningful choices when possible. Discipline involves teaching and it is ongoing, not something that is done just when the child misbehaves. The AAP (2012a, 2015a) suggests three strategies for effective discipline (see also Teaching Guidelines 1.1): Maintaining a positive, supportive, nurturing caregiver-child relationship Using positive reinforcement to increase desirable behaviors Removing positive reinforcements or using punishment to reduce or eliminate undesirable behaviors - *Take Note! The amount of time that a child spends in time-out is typically 1 minute per year of age. For example, a 3-year-old would spend 3 minutes in time-out*

*hyperemesis gravidarum*

Nausea and vomiting during pregnancy severe enough to result in dehydration, acidosis, and weight loss. May require hospitalization; if untreated, can be fatal. - Hyperemesis gravidarum is a severe form of nausea and vomiting of pregnancy associated with significant costs and psychosocial impacts. At least 70% to 85% of women experience nausea and vomiting during their pregnancy. The term morning sickness is often used to describe this condition when symptoms are relatively mild. Studies have shown that nausea and vomiting of pregnancy is associated with improved fetal outcomes, such as lower rates of miscarriage. Such symptoms usually disappear after the first trimester. This mild form mostly affects the quality of life of the woman and her family, whereas the severe form - hyperemesis gravidarum - results in dehydration, weight loss, electrolyte imbalance, and the need for hospitalization. Unlike morning sickness, hyperemesis gravidarum is a complication of pregnancy characterized by persistent, uncontrollable nausea and vomiting that begins in the first trimester and causes dehydration, ketosis, and weight loss of more than 5% of prepregnancy body weight. Risk factors for hyperemesis include previous pregnancy complicated by hyperemesis, molar pregnancies, history of helicobacter pylori infection, multiple gestation, prepregnancy history of genitourinary disorders, clinical hyperthyroid disorders, and prepregnancy psychiatric diagnosis (King et al., 2015). Hyperemesis (uncontrollable vomiting) is estimated to occur in approximately 2% of pregnant women. The prevalence increases in molar pregnancies and multiple gestations. The peak incidence is at 8 to 12 weeks of pregnancy, and symptoms usually resolve by week 20 - *Take Note! Every pregnant woman needs to be instructed to report any episodes of severe nausea and vomiting or episodes that extend beyond the first trimester.*

fetal mortality rate

Number of fetal deaths divided by the sum of live births and fetal deaths occurring in the same population during the same time period. - or fetal death rate refers to the spontaneous intrauterine death of a fetus at any time during pregnancy per 1,000 live births. Fetal deaths later in pregnancy (>20 weeks of gestation) are also referred to as stillbirths. Fetal mortality may be attributable to maternal factors (e.g., malnutrition, disease, or preterm cervical dilation) or fetal factors (e.g., chromosomal abnormalities or poor placental attachment). Fetal mortality is a major, but often overlooked, public health problem. Fetal mortality refers to spontaneous intrauterine death at any time during pregnancy. The fetal mortality rate in the United States is 6.2 per 1,000 live births (CDC, 2015c). Healthy People 2020's goal is to reduce it to 5.6 fetal deaths (USDHHS, 2015). Much of the public concern regarding reproductive loss has concentrated on infant mortality, as less is known about fetal mortality. However, the impact of fetal mortality on United States families is considerable, as it provides an overall picture of the quality of maternal health and prenatal care.

*mgmt for severe preeclampsia*

Severe preeclampsia may develop suddenly and bring with it high blood pressure of more than 160/110 mm Hg, proteinuria of more than 5 g in 24 hours, oliguria of less than 400 mL in 24 hours, cerebral and visual symptoms, and rapid weight gain. This clinical picture signals severe preeclampsia, and immediate hospitalization is needed. Treatment is highly individualized and based on disease severity and fetal age. Birth of the infant is the only cure, because preeclampsia depends on the presence of trophoblastic tissue. Therefore, the exact age of the fetus is assessed to determine viability. Severe preeclampsia is treated aggressively because hypertension poses a serious threat to mother and fetus. The goal of care is to stabilize the mother-fetus dyad and prepare for birth. Therapy focuses on controlling hypertension, preventing seizures, preventing long-term morbidity, and preventing maternal, fetal, or newborn death (Foo et al., 2015). Intense maternal and fetal surveillance starts when the mother enters the hospital and continues throughout her stay. The woman in labor with severe preeclampsia typically receives oxytocin to stimulate uterine contractions and magnesium sulfate to prevent seizure activity. Oxytocin and magnesium sulfate can be given simultaneously via infusion pumps to ensure both are administered at the prescribed rate. Magnesium sulfate is given intravenously via an infusion pump. A loading dose of 4 to 6 g is given over 5 minutes. Then, a maintenance dose of 2 g/hr is given. The client is evaluated closely for magnesium toxicity. If at all possible, a vaginal delivery is preferable to a cesarean birth for better maternal outcomes and less risk associated with a surgical birth. PGE2 gel may be used to ripen the cervix. A cesarean birth may be performed if the client is seriously ill. A pediatrician/neonatologist or neonatal nurse practitioner should be available in the birthing room to care for the newborn. A newborn whose mother received high doses of magnesium sulfate needs to be monitored for respiratory depression, hypocalcemia, and hypotonia. Decreased fetal heart rate variability may occur but, in general, magnesium sulfate does not pose a risk to the fetus. The newborn may exhibit respiratory depression, loss of reflexes, muscle weakness, and neurologic depression

*intervening for preeclampsia*

The woman with mild preeclampsia requires frequent monitoring to detect changes because preeclampsia can progress rapidly. Instruct all women in the signs and symptoms of preeclampsia and urge them to contact their health care professional for immediate evaluation should any occur. Typically, women with mild preeclampsia can be managed at home if they have a good understanding of the disease process, blood pressure and vital signs are stable, there are no abnormal laboratory test results, and if good fetal movement is demonstrated (Teaching Guidelines 19.2). The home care nurse makes frequent visits and follow-up phone calls to assess the woman's condition, to assist with scheduling periodic evaluations of the fetus (such as nonstress tests), and to evaluate any changes that might suggest a worsening of the woman's condition. - Early detection and management of mild preeclampsia is associated with the greatest success in reducing progression of this condition. As long as the client carries out the guidelines of care as outlined by the health care provider and remains stable, home care can continue to maintain the pregnancy until the fetus is mature. If disease progression occurs, hospitalization is required.

*laboratory and dgx testing for preeclampsia*

Various laboratory tests may be performed to evaluate the woman's status. Typically these include a CBC, serum electrolytes, BUN, creatinine, and hepatic enzyme levels. Urine specimens are checked for protein; if levels are 1 to 2+ or greater, a 24-hour urine collection is completed.

Rh incompatibility

exposure of Rh-negative mother to Rh-positive fetal blood; sensitization; antibody production; risk increases with each subsequent pregnancy and fetus with Rh-positive blood Nursing Assessment: maternal blood type and Rh status Nursing Management: RhoGAM at 28 weeks Only will give moms who are Rh- have the potential of getting RhoGam even if she has a miscarriage she will still receive RhoGam Rh+ will NEVER get Rhogam

*client rights*

a way to promote the value and dignity of the client - this information is updated periodically - Most health care agencies and professional organizations have developed some type of document that addresses client rights - Ensuring that client rights are upheld is a key aspect in the care of any client. For the pregnant woman, two clients must be considered—the pregnant woman and her fetus. The American Foundation for Maternal and Child Health developed the Pregnant Patient's Bill of Rights to address specific concerns and situations involving the health and well-being of the mother and her fetus. In addition, the U.S. Department of Health and Human Services has issued laws related to protecting the welfare of newborns (the Baby Doe law). A child, due to his or her age and developmental level, may lack mature decision-making abilities. Many pediatric institutions have adopted a bill of rights for children's health care specific to that institution. This might include the following rights: To be called by name To receive compassionate health care in a careful, prompt, and courteous manner To know the names of all providers caring for the child To have basic needs met and usual schedules or routines honored To make choices whenever possible To be kept without food or drink when necessary for the shortest time possible To be unrestrained if able To have parents or other important persons with the child To have an interpreter for the child and family when needed To object noisily if desired To be educated honestly about the child's health care To be respected as a person (not having people talk about the child within earshot unless the child knows what is happening) For all health care providers, to respect the child's confidentiality about his or her illness at all times

*gestational HTN*

category is used in women with nonproteinuric hypertension of pregnancy, in which the pathophysiologic disturbances of the preeclampsia syndrome do not develop before giving birth. Gestational hypertension is a temporary diagnosis for hypertensive pregnant women who do not meet the criteria for preeclampsia (both hypertension and possibly proteinuria) or chronic hypertension (hypertension first detected before the 20th week of pregnancy). - Previously, was known as pregnancy-induced hypertension or toxemia of pregnancy, but these terms are no longer used - can be differentiated from chronic hypertension, which appears before the 20th week of gestation; or hypertension before the current pregnancy, which continues after the woman gives birth.

*gestational trophoblastic disease (GTD)*

comprises a spectrum of neoplastic disorders that originate in the placenta! very extreme - There is abnormal hyperproliferation of trophoblastic cells that normally would develop into the placenta during pregnancy! - encompass *hydatidiform mole (complete and partial)*, invasive mole, gestational choriocarcinoma, placental-site trophoblastic tumor, and epithelioid trophoblastic tumor - Gestational tissue is present, but the pregnancy is not viable! - The incidence is hard to determine due to uncommon diagnosis and inaccuracy of documentation of pregnancy loss, but it is thought to occur in about *1 in 1,000 pregnancies in the US*; in Asian countries, the rate is 1 of every 120 - the 2 most common types of GTD are hydatidiform mole (partial or complete) and choriocarcinoma -Most women with a classic complete mole present with vaginal bleeding, anemia, excessively enlarged uterus, *preeclampsia*, and hyperemesis.

*Nursery Nurse - provides care to newborns*

i. Administering treatments, medications, tests, and procedures ii. Assisting physicians iii. Creating or updating patient care plans iv. Educating patients' families about proper home and follow-up care v. Maintaining patient records vi. Because they tend to newborns and infants, nursery nurses must often provide non-medical care, such as changing soiled diapers or soothing colicky babies. Other duties depend on the nursery level in which they work. vii. Level I Nursery: In this setting, nursery nurses care for newborns who are healthy and born without complications. Because more hospitals encourage mothers to room-in with their babies, many nursery nurses who once tended to Level I nurseries now provide in-room care. viii. Level II Nursery: In a Level II nursery, nursery nurses provide intermediate care to newborns facing an illness or medical complication, such as premature birth. Because their patients are a bit higher risk than those in Level I nurseries, Level II nursery nurses may provide special care, including oxygen supplementation, IV administration, and more. ix. Level III Nursery: In a Level III nursery, nursery nurses care for newborns battling more serious medical conditions. Patients include small, premature, or potentially ill babies. Nursery nurses working in Neonatal Intensive Care Units must typically learn to use special equipment, such as ventilators and incubators.

h. BOX 1-3 Examples of us Specialty Practice and Special Interest Nursing Organizations

i. American Academy of Nurse Practitioners ii. American Assembly for Men in Nursing iii. American Association for the History of Nursing iv. American Association of Nurse Attorneys v. American Holistic Nurses' Association vi. Association of Nurses in AIDS Care vii. American Association of Critical Care Nurses viii. Dermatology Nurses Association ix. Hospice Nurses Association x. Oncology Nurses Society xi. Sigma Theta Tau International xii. Transcultural Nursing Society

*Postpartum Nurse - provides care to patients who have recently delivered*

i. Mother-Baby Nurses educate and assist new mothers with physical and emotional ii. needs during the post partum period, while in the hospital setting. They perform the dual role of caring for the iii. infants and educating mothers about care. iv. Mother-Baby Nurses provide direct and individualized nursing care to patients based on the application of v. scientific nursing principles. In addition to general nursing care, responsibilities of Mother-Baby Nurses include vi. (but are not limited to): vii. Consults and coordinates with health care team members to assess, plan, implement and evaluate viii. patients' (both mother and infant) care plans ix. Maintains a comfortable, safe and clean environment for mother and infant x. Assists mother with her hygienic needs xi. Checks mother for any post partum bleeding xii. Examines mother's fundus xiii. Inspects c-section incision (if applicable) xiv. Assesses mother's pain, if any xv. Performs emotional status checks on mother, recognizing symptoms such as those related to post partum xvi. depression xvii. Monitors bonding between mother and infant xviii. Takes mother's and infant's vital signs xix. Measures input/output of mother and infant xx. Monitors mother and infant and reports adverse reactions to any treatments or medications being given xxi. Provides mother with breast-feeding tips xxii. Educates mother on sore nipple management xxiii. Teaches mother bathing tips for infant xxiv. Instructs mother on diaper changing xxv. Weighs infant xxvi. Performs heel sticks for bg's of infant xxvii. Keeps track of all feeds and diapers xxviii. Bathes infant xxix. Involves and educates family/significant others in implementing best practices for mother and infant care xxx. General Healthcare Resources, Inc. is an Equal Opportunity Employer xxxi. Records all care information concisely, accurately and completely, in a timely manner, in the appropriate xxxii. format and on the appropriate forms xxxiii. Performs other position-related duties as assigned, depending on assignment setting xxxiv. Nurses in postpartum perform a lot of assessments on new moms to make sure they are healing and recovering properly after giving birth. This may involve checking cesarean incisions or monitoring vital signs. Nurses also remove catheters, which may have been placed during a C-section, or change dressings. They also give pain medication and antibiotics as needed. In addition to caring for the mother, postpartum nurses also monitor babies to make sure they are healthy. xxxv. Besides monitoring the recovery and health of mom and baby, postpartum nurses often provide education. Caring for a brand new little person can be a bit overwhelming, especially for new parents who do not have any experience with babies. Postpartum nurses may instruct new parents on all aspects of taking care of their baby. For example, nurses may teach new moms how to breastfeed, burp or bathe their new baby. xxxvi. Postpartum nurses assist new mothers with physical and emotional needs immediately after the delivery of a baby. As expected of other health professionals, postpartum nurses must follow the policies, philosophy and goals of the health facility and department. A postpartum nurse's role also involves the understanding of what the new mother is experiencing and the ability to provide the proper education and treatment, through observation, resources and consistency. xxxvii. The day-to-day activities of a postpartum nurse will vary, depending on the demands, schedule and facility in which the nurse works. For instance, a postpartum nurse may start lactation services for a new patient. This role could shift to assisting the mother in recovering from emotional stress or postpartum depression. In this case, the nurse is familiar with the risk factors and explores treatment options. The job of a postpartum nurse involves education, support and treatment for new mothers.

*L&D Nurse - provides care to patients in labor who have uncomplicated or complicated deliveries*

i. Providing support for the mother during labor ii. Monitoring the fetal heart rate, and mother's vital signs iii. Measuring the strength and timing of contractions iv. Administering medications and performing diagnostic tests v. Consulting with physicians and other members of the care team vi. Assisting with inducing labor vii. Coaching the mother during delivery viii. Identifying and assisting with complications ix. Assisting with various procedures, including Cesarean sections x. Monitoring and performing tests on newborns xi. Providing education and support to mothers and families after delivery xii. The labor and delivery nurse works in a clinic, hospital or birth center setting, assisting the physician or midwife with all aspects of pregnancy and childbirth. Nursing duties during the birth itself include monitoring fetal heartbeat and the length and strength of contractions, as well as administering medication, coaching the woman and assisting with any complications that arise. Following the birth, they perform tests on the newborn and provides guidance to the mother on all aspects of recovery and infant care. The duties of an L&D nurse may include seeing expectant mothers in their prenatal visits, providing family support and teaching childbirth preparation classes.

*HIPAA*

i. Right to Privacy - Unnecessary exposure of the woman's body ii. Cultural considerations iii. Private adoptions iv. Protection by law=HIPAA Delegation 1. Clearly defined and thoroughly described v. Repetitive and represent routine care vi. Have a predictable outcome vii. Require little or no modification from one patient care situation to another viii. Involve no on-going assessment and no significant interpretation ix. Be within the training and competence of the assistive personnel - the confidentiality of health care information is now mandated by law. - The primary intent of the law is to maintain health insurance coverage for workers and their families when they change or lose jobs - Another aspect of the law requires the Department of Health and Human Services to establish national standards for electronic transmission of health information - With the increased use of electronic medical records (EMRs) and electronic billing came the increased possibility that personal health information might be inappropriately distributed. Client confidentiality and privacy must be maintained in the same manner as it is with paper documentation. Nurses can ensure that privacy is maintained when using computerized documentation and EMRs by doing the following: 1. Always maintain the security of personal log-in information; nurses should never share it with other health care providers or other persons. 2. Always log off when leaving the computer. 3. Do not leave client information visible on a monitor screen when the computer/monitor is unattended. 4. Use safeguards, such as encryption, when using alternative means of communication such as email. xi. Exceptions to confidentiality exist. For example, suspicion of physical or sexual abuse and injuries caused by a weapon or criminal act must be reported to the proper authorities. Abuse cases are reported to the appropriate welfare authorities, whereas criminal acts are reported to the police. In addition, if the minor is a threat to self, then information may need to be disclosed to protect the child. The health care provider must also follow public health laws that require reporting certain infectious diseases to the local health department (e.g., tuberculosis, hepatitis, HIV, and other sexually transmitted infections). Finally, there is a duty to warn third parties when a specific threat is made to an identifiable person. xii. Health care providers must strike a balance between confidentiality and required disclosure. Even if disclosure is required it is recommended that the health care provider attempt to gain consent for the disclosure and when possible inform the minor of the limits to confidentiality and consent prior to the initiation of care

*maternal-fetal conflict*

in maternity nursing, the ethical principles of beneficence and autonomy provide the fundamental framework that guides the management of all pregnant women - as there is the need for consideration of the fetus, autonomy can become a complex issue, which gives rise to this Fetal care becomes problematic when what is required to benefit one member of the dyad will cause an unacceptable harm to the other. When a fetal condition poses no health threat to the mother, caring for the fetal client will always carry some degree of risk to the mother, without direct therapeutic benefit for her. The ethical principles of beneficence ("be of benefit") and nonmaleficence ("do no harm") can come into conflict. Because the clients are biologically linked, both, or neither, must be treated alike. It would be unethical to recommend fetal therapy as if it were medically indicated for both clients. Still, given a recommendation for fetal therapy, pregnant women, in most cases, will consent to treatment, which promotes fetal health. When pregnant women refuse therapy, health care providers must remember that the ethical injunction against harming 1 client in order to benefit another is virtually absolute. The use of court orders to force treatment on pregnant women raises many ethical concerns. *Court orders* force pregnant women to forfeit their autonomy in ways not required of competent men or nonpregnant women. There is an inconsistency in allowing competent adults to refuse therapy in all cases but pregnancy. The American College of Obstetricians and Gynecologists (ACOG) advocates counseling and education to convince a mother to follow her doctor's advice and condemns the use of coercion on a pregnant woman, as this violates the intent of the informed consent process. Faced with a continuing disagreement with a pregnant woman, a physician should turn to an institutional ethics committee. Resorting to the legal system is almost never justified

*Nurse Practice Acts!!!* how it affects nursing practice, and the legalities of practicing using these standards

individual states are responsible for legally determining the scope of practice for nurses - law established to regulate nursing practice - laws established in each state to regulate the practice of nursing - They are broadly worded and vary among states, but all of them have certain elements in common, such as the following: 1. Protect the public by defining the legal scope of nursing practice, excluding untrained or unlicensed people from practicing nursing. 2. Create a state board of nursing or regulatory body having the authority to make and enforce rules and regulations concerning the nursing profession. 3. Define important terms and activities in nursing, including legal requirements and titles for RNs and LPNs. 4. Establish criteria for the education and licensure of nurses. - the board of nursing for each state has the legal authority to allow graduates of approved schools of nursing to take the licensing examination - those who successfully meet the requirements for licensure are then given a license to practice nursing in the state. - The license, which must be renewed at specified intervals, is valid during the life of the holder and is registered in the state. Many states have a requirement for a specified number of continuing education units to renew and maintain licensure. There are two ways in which nurses can practice in a state other than in the one they were originally licensed. One is by reciprocity, which allows a nurse to apply for and be endorsed as a registered nurse by another state. There are also 24 states that are members of the Nurse Licensure Compact, allowing a nurse who is licensed and permanently lives in one of the member states to practice in the other member states without additional licensure - The license and the right to practice nursing can be denied, revoked, or suspended for professional misconduct (e.g., incompetence, negligence, chemical impairment, or criminal actions). iv. As nursing roles continue to expand and issues in nursing are resolved, revised nurse practice acts will reflect those changes. All nurses must be knowledgeable about the specific nurse practice act for the state in which they practice.

*delegation*

involves the transfer of responsibility for the performance of an activity to another individual while retaining accountability for the outcome. i. Used appropriately, delegation can result in safe and effective nursing care. Delegation can free the registered nurse (RN) to focus on assessment and development or revision of the nursing care plan. Delegation allows the RN to attend to more complex patient care needs, develops the skills of nursing assistive personnel, and promotes cost containment for the health care organization. The RN determines appropriate nursing practice by using nursing knowledge, professional judgment, and the legal authority to practice nursing ii. In delegating, the RN must ensure appropriate assessment, planning, implementation, and evaluation. Decision making about delegation is a continuous process. These guidelines provide a quick reference for the delegation decision-making information found in each skill. iii. Look at delegation handout on moodle

*determine what malpractice is and an action plan to prevent a legal action against yourself as a nurse*

is negligence on the part of the nurse. it is determined if the nurse owed a duty (care) to the client/patient and did not carryout the duty (care) and the client/patient was injured. - act of negligence as applied to a professional person such as a physician, nurse, or dentist - nurses have more responsibility today than in the past, which opens the door to greater legal risk. - legal Considerations must remain very cognizant of the scope of practice, standards, nurse practice act, and policies that direct nursing care for this population of patients. - obstetrics considered the financial loss leader - Moment of intrauterine insult/injury is difficult - Loss of "perfect" - Parental expectations - Contributing Factors: i. Negligence/Incompetence, Accident, Poor judgment, Miscommunication, Staffing i. common categories of malpractice claims: i. Failure to follow standards of care: for example, you fail to follow the standards for administering insulin or other injectable medications ii. Failure to use equipment in a responsible manner: for example, you attempt to use a bariatric patient lift for the first time without getting help and the patient falls iii. Failure to assess and monitor: for example, you fail to follow your hospital's standards for postoperative assessments after receiving a patient from the operating room and response to a ruptured suture line is delayed iv. Failure to communicate: for example, you fail to communicate your concerns about an elderly patient being discharged home; she lives alone and is soon rehospitalized because no provisions were made to secure the nursing care she needed after discharge v. Failure to document: for example, you work up the chain of command when your repeated calls to a physician to see a patient that you believe is in danger of arresting are ignored. Before any physician sees the patient, he arrests and, despite a code, dies. You document the arrest, code, and death but fail to document all the steps you took to get the patient the medical attention he needed. Sixteen months later, the family sues and you try to remember what action you took that evening—most of which was never recorded. vi. Failure to act as a patient advocate or to follow the chain of command: for example, you are in the operating room and watch a surgeon break the sterile field twice. No one else seems to notice. You are intimidated by this surgeon and fail to bring this to anyone's attention. You learn that the patient developed a serious infection postoperatively.

family structure

is the composition of people who interact with one another on a regular, recurring basis in socially sanctioned ways. It involves how the family unit is organized, which often influences the relationship of the members of the family. Family members can be gained or lost through events such as divorce, marriage, birth, death, abandonment, and incarceration. All of these events can alter the family structure, leading to roles being redefined or redistributed. Table 1.2 provides examples of the types of family structures found in today's society. The traditional nuclear family is no longer considered the dominant family structure. From 1980 to 2012, the percentage of children who were living with their own married parents decreased steadily from 77% to 64% (FIFCFS, 2013). By understanding clients' family structure and any changes that may occur, nurses can provide them with support to alleviate or prevent dysfunctional alterations in family coping and adaptation. - *Take Note! The lifestyle of the parents basically is the lifestyle of the children. For instance, parents who are inactive and eat poorly will have children who do the same, and the problems associated with these unhealthy habits, such as diabetes, obesity, and early heart disease, are showing up earlier in children and adolescents. It is important for parents to serve as role models for proper nutrition and physical activity (through sports, hobbies, or other activities).*

childhood mortality rate

is the number of deaths per 100,000 population in children 1 to 14 years of age. The childhood mortality rate in the United States has decreased significantly since 1980 but disparities by gender, age, race, and ethnicity persist (Child Trends DataBank, 2015a). The current mortality rate for children between ages 1 and 4 years is 25.5 per 100,000, with the leading cause of death being unintentional injuries followed by congenital malformations (CDC/NCHS, 2015). The mortality rate for children of ages 5 to 14 years is 13 per 100,000, with the leading cause being unintentional injuries followed by cancer (CDC/NCHS, 2015). Other causes of childhood mortality include suicide, homicide, diseases of the heart, influenza, and pneumonia. Even as research continues into the preventable nature of childhood injuries, unintentional injury (e.g., motor vehicle accidents, fires, drowning, bicycle or pedestrian accidents, poisoning, falls) remains a leading cause of mortality and morbidity in children. These injuries have far-reaching consequences for children, families, and society in general. Risk factors associated with childhood injuries include young age, male gender, low socioeconomic status, parents who are unmarried or single, low maternal education level, poor housing, parental drug or alcohol abuse, and low support within the family. These deaths can often be prevented through education about the value of using car seats and seat belts, the dangers of driving under the influence of alcohol and other substances, and the importance of pedestrian and bicycle safety, fire safety, water safety, and home safety. - *Take Note! In the United States, American Indian/Alaska Natives children, followed by African American children, have the highest unintentional injury death rate*

*scope of practice!!!* how it affects nursing practice, and the legalities of practicing using these standards

limits of nursing practice set forth in state statutes. - defines activities that are specific and unique to nursing - Standards allow nurses to carry out professional roles, serving as protection for the nurse, the patient, and the institution where health care is provided - Each nurse is accountable for his or her own quality of practice and is responsible for the use of these standards to ensure knowledgeable, safe, and comprehensive nursing care.

*Garret's Law*

mandatory for health care officials to contact authorities if any baby is born with an illegal substance in their system or if any child develops a health care problem that is drug related. - the mothers of newborn infants who test positive for illegal drugs are presumed to be guilty of parental neglect under the state's civil code, and medical personnel can report them to police and child protective service workers. - Act 1176 of 2005 (Garrett's Law) reads, "this act is necessary to clarify the law so that the Arkansas State Police Child Abuse Hotline can accept reports of this nature and so that the newborn children can be provided services to protect their health and safety."

neonatal mortality rate

neonatal death: The death of a live newborn during the first 28 days of life. An early neonatal death is considered by the World Health Organization to be a death within the first 7 days of life. - is the number of infant deaths occurring in the first 28 days of life per 1,000 live births. The United States now ranks 41st in the world in terms of neonatal mortality, the death rate of infants less than 1-month old. The neonatal mortality rate is 4.5 per 1000 live births (World Bank, 2015c). Healthy People 2020's goal is to reduce it to 4.1 per 1000 live births (USDHHS, 2015). Each year the deaths of 2 million babies are linked to complications during birth or within the first month. Furthermore, the burden is inequitably carried by the poor. Evidence-based strategies are urgently needed to reduce the burden of intrapartum-related deaths and effective interventions to address these problems must be incorporated into policy decisions (Oza et al., 2015). The reliability of the neonatal mortality estimates depends on accuracy and completeness of reporting and recording of births and deaths. Underreporting and misclassification are common, especially for deaths occurring early on in life. Perinatal mortality encompasses late and early neonatal mortality and is defined as the number of stillbirths and deaths in the first week of life per 1,000 live births; it is also a useful indicator. The perinatal mortality is the sum of the fetal mortality and the neonatal mortality. Work is ongoing to improve estimates of stillbirth rates, a major component of perinatal mortality

resilience

refers to the qualities that enable a person to cope with significant adverse events or stresses and still function competently - Exposure to stress is not limited to disasters or traumatic events, however. Stress can also include areas such as inadequate finances, family crises such as divorce, inadequate support systems, illness, or violence. Stress can also be associated with the normal problems associated with growth and development, such as entering a new classroom, learning a new skill, or being teased by a classmate. Similar to disasters and traumatic events, the effects of these stressors can dramatically affect the health status of a woman, child, or family.

Affordable Care Act (ACA)

seeks to reform the current U.S. health care system so that all Americans have access to quality, affordable health care; improved access to innovative and preventive health care programs and therapies; and expanded insurance coverage; seeks to establish a more integrated health care system - is expected to expand insurance options for people with disabilities and make them more affordable, people with pre-existing chronic conditions and disabilities can no longer be dropped by insurance companies - will require private insurers to cover preventative health care services for women w/o cost sharing such as copayments, deductibles, or co-insurance - EBP screening and counseling for depression, diabetes, cholesterol, obesity, various cancers, HIV, STIs are also covered - A range of women's services will also be covered w/o cost sharing, include: annual well woman visits, support for breast-feeding, counseling and screening for intimate partner violence, all FDA-approved contraception methods, and mammograms

Scrub Nurse

works directly with the surgeons during a cesarean delivery by passing instruments, etc, to the physicians.


Ensembles d'études connexes

Chapter 20 The Imperial republic

View Set

spanish test on future and future perfect

View Set

Chapter 8 Review: Statistics and Multiplication Equations

View Set

Chapter 7: Managing Change & Disruptive Innovations

View Set